You are on page 1of 301

0:00:07/03:00:00

A 22-year-old man is stabbed in the leg after a fight and sustains a significant laceration
leading to damage to the femoral artery. Paramedics on the scene estimate that he has lost
around 1.2 litres of blood.

A Decreased respiratory rate

B Decreased urine output

C Delayed capillary refill

D Hypotension

E Tachycardia

Explanation 

E Tachycardia

This patient has moderate blood loss, (between 750 and 1500ml). In this situation cardiac
output cannot be maintained by vasoconstriction alone, therefore an increase in pulse rate is
seen.

A Decreased respiratory rate

Decreased respiratory rate is incorrect. In this situation an increased respiratory rate to


greater than 20 breaths per minute is usually seen.

B Decreased urine output

Decreased urine output is incorrect. Although there is a fall in urine output to 20-30ml/hr,
this occurs later, (after an initial increase in pulse rate).

C Delayed capillary refill


Delayed capillary refill is incorrect. Delayed capillary refill is seen, but it occurs later, after
tachycardia.

D Hypotension

Hypotension is incorrect. The increased pulse rate is usually adequate to maintain blood
pressure, systolic blood pressure is therefore maintained and a small increase in diastolic
blood pressure is seen.
41024

Rate this question:

Next Question

Previous Question Tag Question

Feedback End Review

Difficulty: Average

Peer Responses %

Q. Answered Flagged

Q1

Q2

Q3

Q4

Q5

Q6

Q7
Q. Answered Flagged

Q8

0:00:07/03:00:00

A 38-year-old South Asian woman who has been taking long-standing Phenytoin therapy
for epilepsy comes to the Rheumatology Clinic for review. Over the past few months she
has suffered from increasing lethargy, bony pains, and problems climbing the stairs and
getting up from a low chair. She takes no regular medication apart from the phenytoin. On
examination her BP is 122/82 mmHg; pulse is 77/min and regular. Her BMI is 27. There is
proximal myopathy but no other abnormal findings.

Investigations;

Hb 11.0 g/dl

WCC 7.2 x109/l

PLT 203 x109/l

Na+ 137 mmol/l

K+ 4.2 mmol/l

Creatinine 95 micromol/l

Calcium 2.05 mmol/l

ALP 189 U/l

PO4- 0.70 mmol/l

A Hypoparathyroidism

B Osteomalacia

C Osteoporosis

D Paget’s disease

E Primary hyperparathyroidism

Explanation 
B Osteomalacia

Long term use of Phenytoin is recognised to be associated with osteomalacia, and this
patient, being of South Asian heritage is at increased risk of the condition. The low calcium,
coupled with raised alkaline phosphatase and decreased phosphate are consistent with the
diagnosis. Calcium and Vitamin D supplementation is the intervention of choice.

A Hypoparathyroidism

Hypoparathyroidism is incorrect. In hypoparathyroidism, phosphate levels are elevated.

C Osteoporosis

Osteoporosis is incorrect. Osteoporosis is associated with reduced bone mineral density,


rather than with hypocalcaemia.

D Paget’s disease

Paget’s disease is incorrect. This is associated with raised alkaline phosphatase in the
presence of normal calcium.

E Primary hyperparathyroidism

Primary hyperparathyroidism is incorrect. Hyperparathyroidism is associated with


hypercalcaemia and hypophosphataemia.
41158

Rate this question:

Next Question

Previous Question Tag Question

Feedback End Review

Difficulty: Average

Peer Responses %
Q. Answered Flagged

Q1

Q2

Q3

Q4

Q5

Q6

Q7

Q8

Q9

0:00:07/03:00:00

A 44-year-old man comes to the Gastroenterology Clinic for review. He has paid privately
for a colonoscopy after two of his relatives were diagnosed with colon cancer, and is
extremely concerned to reduce future risk of colonic carcinoma as much as possible.
Examination in the clinic is unremarkable, abdomen is soft and non-tender, and his BMI is
24. The colonoscopy was reported as entirely normal. He asks about dietary risk reduction
for colon cancer.

A Vitamin A

B Vitamin B12

C Vitamin C

D Vitamin D

E Vitamin E

Explanation 

D Vitamin D

Vitamin D has been shown in in vitro models to positively affect factors that are linked both
to colon cancer initiation and progression. Observational studies suggest that lower levels
of vitamin D are associated with higher incidence of colon cancer, and patients who have
lower post-operative levels of vitamin D after colonic resection have a poorer prognosis
with respect to cancer progression. Intervention data on the effect of vitamin D with
respect to cancer prevention is conflicting, and further studies are ongoing.

A Vitamin A

Vitamin A is incorrect. Vitamin A supplementation may be associated with increased risk of


lung cancer and heart disease.

B Vitamin B12
Vitamin B12 is incorrect. Vitamin B6 (pyridoxine) may be associated with decreased risk of
colon cancer, not vitamin B12.

C Vitamin C

Vitamin C is incorrect. No positive effect of vitamin C supplementation has been seen on


risk of colon cancer.

E Vitamin E

Vitamin E is incorrect. Vitamin E supplementation may increase the risk of prostate cancer,
but it has no effect in reducing colon cancer risk.
40965

Rate this question:

Next Question

Previous Question Tag Question

Feedback End Review

Difficulty: Average

Peer Responses %

Q. Answered Flagged

Q1

Q2

Q3

Q4
Q. Answered Flagged

Q5

Q6

Q7

Q8 
0:00:07/03:00:00

A 59-year-old man with a history of Type 2 diabetes for the past 3 years presents to the
Emergency Department with a 4th episode of gout affecting his right great toe in the last 5
months. His uric acid is measured at 0.56 mmol/l. On two previous occasions he was treated
with two courses of diclofenac, and on the last occasion a short course of prednisolone.
Medication for diabetes includes metformin, sitagliptin, and Lisinopril for renoprotection. On
examination his BP is 138/90 mmHg; pulse is 80/min and regular. BMI is 32. There is
erythema and pain over the right great toe.

Investigations;

Hb 12.9 g/dl

WCC 11.0 x 109/l

PLT 181 x 109/l

CRP 145 mg/l

Na+ 137 mmol/l

K+ 4.9 mmol/l

Creatinine 112 micromol/l

HbA1c 66 mmol/mol (8.2%)

A Allopurinol

B Continuous low dose Prednisolone

C Diclofenac for acute attacks

D Colchicine

E Prednisolone for acute attacks

Explanation 

A Allopurinol
In this situation, Allopurinol over the long term, (after a short course of non-steroidals or
Colchicine), is the optimal way to manage this patient’s gout. As well as reducing uric acid
long term, and hence the risk of gout exacerbations, Allopurinol has also shown a trend
with respect to reducing progression of diabetic nephropathy. It should be started after the
acute episode of gout has subsided, at a dose of 100mg per day, up titrated to bring uric
acid within target (0.36 mmol/l).

B Continuous low dose Prednisolone

Continuous low dose Prednisolone is incorrect. The adverse event (AE) profile of long-term
corticosteroid use is not favourable with respect to the treatment of gout.

C Diclofenac for acute attacks

Diclofenac for acute attacks is incorrect. Diclofenac runs the risk of a negative short-term
impact on blood pressure and fluid retention, particularly in a patient with Type 2 diabetes.

D Colchicine

Colchicine is incorrect. Low dose Colchicine is an option for gout prophylaxis, although it is
usually considered a second line option because of diarrhoea.

E Prednisolone for acute attacks

Prednisolone for acute attacks is incorrect. It is likely to have too great a negative impact on
blood pressure, fluid retention, glucose control and bone mineral density.
41012

Rate this question:

Next Question

Previous Question Tag Question

Feedback End Review

Difficulty: Average

Peer Responses %
Q. Answered Flagged

Q1

Q2

Q3

Q4

Q5

Q6

Q7

Q8

Q9

0:00:07/03:00:00

A 76-year-old woman is admitted to the Emergency Department with confusion, malaise,


nausea and vomiting. She has recently been reviewed by her GP who diagnosed her with
heart failure and atrial fibrillation, and prescribed a number of medications. She refused
admission to hospital and insisted he manage her at home. On examination her BP is 110/70
mmHg, pulse is 38/min, (slow atrial fibrillation). There are bibasal crackles on auscultation of
the chest, and bilateral pitting oedema of both ankles. She complains of decreased visual
acuity and says that things look yellow.

Investigations:

Hb 13.1 g/dl

WCC 7.0 x109/l

PLT 189 x109/l

Na+ 137 mmol/l

K+ 5.2 mmol/l

Creatinine 231 micromol/l

Glucose 5.4 mmol/l

A Atorvastatin

B Bisoprolol

C Clopidogrel

D Digoxin

E Ramipril

Explanation 

D Digoxin
Digoxin toxicity results in nausea, lethargy, yellowing of vision and symptomatic
bradycardia. It is likely, given that her creatinine is 231 micromol/l, that either her renal
function has deteriorated as a result of other medicine prescribed by the GP; or that she
has become dehydrated because of ongoing confusion coupled with not eating and
drinking. The Digoxin should be discontinued.

A Atorvastatin

Atorvastatin is incorrect. Atorvastatin is recognised to cause nightmares, it is not usually


associated with confusion.

B Bisoprolol

Bisoprolol is incorrect. It is associated with confusion, lethargy and bradycardia, but not with
yellowing of the vision.

C Clopidogrel

Clopidogrel is incorrect. Headache and dizziness but not confusion was reported in the
clopidogrel clinical trial program.

E Ramipril

Ramipril is incorrect. ACE inhibition may have contributed to any rise in creatinine, and is
reported to cause dizziness, headache and loss of sense of taste, but is not associated with
confusion.
41039

Rate this question:

Next Question

Previous Question Tag Question

Feedback End Review

Difficulty: Average

Peer Responses %
Q. Answered Flagged

Q1

Q2

Q3

Q4

Q5

Q6

Q7

Q8

Q9

0:00:07/03:00:00

A 61-year-old man who is treated for Type 2 diabetes with oral Acarbose and Metformin, and
long-acting subcutaneous insulin, is admitted to the Emergency department after a
hypoglycaemic episode. Although he initially recovered with a sugary drink given at the
scene, he has deteriorated again during the ambulance journey. He is confused and
disorientated on arrival in the Emergency unit. His BP is 132/82 mmHg, pulse is 95/min and
regular and he is bathed in sweat. Finger stick glucose is recorded at 1.4 mmol/l. He has been
cannulated by the ambulance staff.

A IM glucagon

B 5% IV glucose

C 20% IV glucose

D 50% IV glucose

E Oral sucrose

Explanation 

C 20% IV glucose

This patient has life-threatening hypoglycaemia, and IV glucose is the most efficient way to
raise is blood glucose in a short time. Out of the options given, 20% glucose solution
delivers a significant carbohydrate load without the problems of marked viscosity and
hypertonicity associated with 50% solution. 80ml is usually given from a 100ml bag over
approximately 15 mins.

A IM glucagon

IM glucagon is incorrect, although this is the default option in the absence of IV access.

B 5% IV glucose
5% IV glucose is incorrect. This does not deliver a significant carbohydrate load to correct
hypoglycaemia.

D 50% IV glucose

50% IV glucose is incorrect. The viscosity and marked hypertonicity of 50% glucose solution
mean that 20% solution is the preferred option.

E Oral sucrose

Oral sucrose is incorrect. Acarbose blocks metabolism of sucrose, it is therefore not an


option in patients taking oral acarbose.
41093

Rate this question:

Next Question

Previous Question Tag Question

Feedback End Review

Difficulty: Average

Peer Responses %

Q. Answered Flagged

Q1

Q2

Q3

Q4
Q. Answered Flagged

Q5

Q6

Q7

Q8 
0:00:07/03:00:00

A 17-year-old girl presents with pain and swelling over her right knee that has progressively
worsened over the last 10 days. She is actively involved in sports and has recently been
selected for national championships. Clinical examination reveals pain over the tibial
tubercule, and an X-ray of the knee reveals fragmentation of the tibial tubercule.

A Infrapatellar bursitis

B Osgood-Schlatter’s disease

C Osteochondritis dissecans

D Torn anterior cruciate ligament

E Torn meniscus

Explanation 

B Osgood-Schlatter’s disease

Osgood–Schlatter disease causes pain and swelling over the tibial tubercle. A traction
apophysitis of the patellar tendon, it is seen in sports players undergoing rigorous training.

A Infrapatellar bursitis

Infrapatellar bursitis is incorrect. Infrapatellar bursitis is caused by kneeling (housemaid’s


knee). There is local pain, tenderness and a fluctuant swelling.

C Osteochondritis dissecans

Osteochondritis dissecans is incorrect. Osteochondritis dissecans is more common in


adolescent and young adult males. Locking or giving way of the joint is seen in this condition.

D Torn anterior cruciate ligament


Torn anterior cruciate ligament is incorrect. A torn anterior cruciate ligament allows the
tibia to be pulled forwards on the femur when the knee is flexed to 90°.

E Torn meniscus

Torn meniscus is incorrect. A twisting injury is a common history in meniscal tears. There is
immediate medial or lateral knee pain and dramatic swelling develops within a few hours.
49034

Rate this question:

Next Question

Previous Question Tag Question

Feedback End Review

Difficulty: Difficult

Peer Responses %

Q. Answered Flagged

Q1

Q2

Q3

Q4

Q5

Q6

Q7
Q. Answered Flagged

Q8
0:00:07/03:00:00

A 72-year-old man presents to the Emergency Department for the third time in the last 6
months because of a fall. Apart from a history of hypertension and asthma, he has otherwise
remained well until now, and worked until 4 years earlier as a solicitor. According to a
neighbour he has become progressively more unsteady over the past few months, seems
mildly confused, and is finding it difficult to converse. On examination his BP is 155/95 mmHg
lying, dropping to 125/80 mmHg on standing. He is very unsteady with freezing when you
ask him to walk across the room. There is no resting tremor, although you notice a slight
tremor on movement, worse on the right hand vs the left hand side. There is bradykinesia
and micrographia, and vertical gaze palsy.

A Depression

B Normal pressure hydrocephalus

C Parkinson’s disease

D Pick’s disease

E Progressive supranuclear palsy

Explanation 

E Progressive supranuclear palsy

PSP is suggested by the prominent autonomic dysfunction, loss of vertical gaze, and
prominent bradykinesia and micrographia, in the absence of significant resting tremor.
Criteria for probable PSP have been defined, which include:

Gradually progressive disorder


Onset at age 40 or later
Vertical (upward or downward gaze) supranuclear palsy and prominent postural
instability with falls in the first year of disease onset
No evidence of other disease that could explain these features

A Depression
Depression is incorrect, given that we have prominent features of motor dysfunction and
autonomic dysfunction which cannot be explained by depression alone.

B Normal pressure hydrocephalus

Normal pressure hydrocephalus is incorrect. This is associated with the triad of gait
disturbance, dementia and incontinence.

C Parkinson’s disease

Parkinson’s disease is incorrect. More prominent tremor, less severe postural hypotension,
and no vertical gaze palsy would be expected in idiopathic Parkinson’s disease.

D Pick’s disease

Pick’s disease is incorrect. Pick’s is frontal lobe dementia, associated with early and
prominent mood disturbance and inappropriate behaviour.
41152

Rate this question:

Next Question

Previous Question Tag Question

Feedback End Review

Difficulty: Average

Peer Responses %

Q. Answered Flagged
Q. Answered Flagged

Q1

Q2

Q3

Q4

Q5

Q6

Q7

Q8
0:00:07/03:00:00

A 61-year-old man with a history of Type 2 diabetes comes to the clinic for review. He has
suffered from increasing shortness of breath over the past few months, a decrease in his
exercise tolerance, and he tells you he has been coughing up pink frothy sputum on most
mornings during the past week. He takes a number of medications for control of diabetes
and cardiovascular disease, and has suffered an inferior myocardial infarction some 5 years
earlier. On examination his BP is 132/82 mmHg; pulse is 92/min and regular. He has bilateral
pitting oedema to the mid calves, and bilateral coarse crackles to the mid zones on
auscultation of his chest.

Investigations:

Hb 13.1 g/dl

WCC 8.1 x109/l

PLT 181 x109/l

Na+ 134 mmol/l

K+ 4.9 mmol/l

Creatinine 122 micromol/l

Glucose 7.9 mmol/l

Upper lobe diversion, small left pleural effusion, pulmonary infiltrates consistent
CXR
with heart failure

A Amlodipine

B Basal insulin

C Indapamide

D Metformin

E Pioglitazone

Explanation 
E Pioglitazone

Thiazolidinediones, such as Pioglitazone are recognised to exacerbate fluid retention. They


may worsen underlying heart failure, and for this reason they are not recommended in this
situation. Glitazones are also recognised to decrease bone mineral density, lead to weight
gain over time, and Pioglitazone has been associated with bladder cancer. As such they are
no longer in widespread use for the treatment of Type 2 diabetes.

A Amlodipine

Amlodipine is incorrect. Although dihydropyridine calcium antagonists may exacerbate lower


limb oedema, they do not particularly increase the risk of heart failure.

B Basal insulin

Basal insulin is incorrect. Although insulin does exacerbate salt and water retention, it is
likely to be needed to maintain glycaemic control here.

C Indapamide

Indapamide is incorrect. Indapamide is a thiazide like diuretic and does not worsen the risk of
heart failure.

D Metformin

Metformin is incorrect. Metformin is associated with improved cardiac outcomes, and only
really needs to be discontinued during a period of significant hypotension, or where
significant renal impairment exists, (decreased dose at GFR<60, consider stopping at
GFR<30).
41081

Rate this question:

Next Question

Previous Question Tag Question

Feedback End Review

Difficulty: Average

Peer Responses %
Q. Answered Flagged

Q1

Q2

Q3

Q4

Q5

Q6

Q7

Q8

Q9

0:00:07/03:00:00

An 18-year-old girl presents with asymptomatic skin changes on the left thigh that have
become apparent over the last month. Examination reveals a 5cm annular plaque which feels
very firm and thickened. There is an erythematous border and the slightly hyperpigmented
central skin looks atrophic.

A Erythema chronicum migrans

B Lichen sclerosus

C Morphoea

D Pityriasis rosea

E Pityriasis versicolor

Explanation 

C Morphoea

The cause of morphoea is uncertain.


It typically presents with one or more indurated plaques.
These are usually initially inflamed, especially around the border, leading to an annular
appearance.
As the sclerotic process progresses, plaques go from thickened to atrophic.
Eventually, the inflammation usually subsides and the plaque turns brown or ivory
white.
Some patients describe tightness of the skin, though the condition may be
asymptomatic.
The linear variant of morphoea is more common in children: on the forehead,
this is known as morphoea ‘en coup de sabre’.
Linear morphoea over a joint can result in restriction of movement if not treated
effectively.
Treatment options for morphoea in general include potent or very potent topical
corticosteroids, oral corticosteroids, phototherapy and methotrexate.

A Erythema chronicum migrans

Erythema chronicum migrans is incorrect. This is the initial manifestation of Lyme disease.
There may be a history of tick bite, followed by extending macular erythema that forms an
annular lesion; induration is not a feature.

B Lichen sclerosus

Lichen sclerosus is incorrect. This is an autoimmune condition that usually affects the genital
area; extra-genital lichen sclerosus is unusual. It can cause induration but more commonly
results in atrophic changes, with a texture likened to cigarette paper. It is typically strikingly
hypopigmented.

D Pityriasis rosea

Pityriasis rosea is incorrect. This begins with a ‘herald patch’ that is followed after a few days
by a widespread truncal eruption of plaques with fine peripheral scaling. These are classically
arranged in a Christmas tree configuration; they are superficial and not significantly
indurated.

E Pityriasis versicolor

Pityriasis versicolor is incorrect. This is caused by overgrowth of the (usually) commensal


yeast, Malassezia furfur. It typically presents in young adults in warm weather or after a sunny
holiday, with asymptomatic hypo- or sometimes hyperpigmented macules with very fine
peripheral scaling; the lesions are very superficial rather than indurated.
49002

Rate this question:

Next Question

Previous Question Tag Question

Feedback End Review

Difficulty: Difficult

Peer Responses %
Q. Answered Flagged

Q1

Q2

Q3

Q4

Q5

Q6

Q7

Q8

Q9

0:00:07/03:00:00

A patient with diabetic retinopathy is treated with panretinal photocoagulation in the Eye
Clinic and is followed up in the Diabetic Clinic.

A Visual field constriction

B Vitreous haemorrhage

C Retinal burns

D Optic atrophy

E Tractional retinal detachment

Explanation 

B Vitreous haemorrhage

The source of the vitreous bleeding is likely to be further growth of fragile neovascular
tissue. The role of laser photocoagulation is to reduce the production of vascular endothelial
growth factors caused by ischaemic retina.

A Visual field constriction

Visual field constriction is incorrect. Visual field constriction is a possible sequel of


proliferative diabetic retinopathy treated by laser treatment, but is not in itself an indication
for further laser treatment.

C Retinal burns

Retinal burns is incorrect. Retinal burns are a possible sequel of proliferative diabetic
retinopathy treated by laser treatment, but is not in itself an indication for further laser
treatment.

D Optic atrophy
Optic atrophy is incorrect. Optic atrophy is a possible sequel of proliferative diabetic
retinopathy treated by laser treatment, but is not in itself an indication for further laser
treatment.

E Tractional retinal detachment

Tractional retinal detachment is incorrect. Tractional retinal detachment is a possible sequel


of proliferative diabetic retinopathy treated by laser treatment, but is not in itself an
indication for further laser treatment.
828

Rate this question:

Next Question

Previous Question Tag Question

Feedback End Review

Difficulty: Average

Peer Responses %

Q. Answered Flagged

Q1

Q2

Q3

Q4

Q5

Q6
Q. Answered Flagged

Q7

Q8

0:00:07/03:00:00

A 34-year-old woman presents to the Hepatology Clinic for review. Routine pre-
employment screening has identified her as being positive for hepatitis C infection. She has
no symptoms of note and her only medication is the combined oral contraceptive pill. She
admits to use of intravenous drugs on three occasions during her time in University.

A Age at initiation

B Co-existence of other risk factors for chronic liver disease

C Fibrosis on liver biopsy

D Hepatitis C genotype

E Transaminases

Explanation 

D Hepatitis C genotype

Genotype 1 has the lowest success rates with respect to viral clearance, as such NICE
recommends the newer agents such as Telaprevir and Boceprevir, (NS3/4A serine protease
inhibitors), as initial therapy where this genotype is identified. Where these agents are used,
sustained viral response is seen in over 90% of patients.

A Age at initiation

Age at initiation is incorrect. Although increasing age is associated with increased risk of
fibrotic liver disease, it does not influence use of newer anti-virals.

B Co-existence of other risk factors for chronic liver disease

Co-existence of other risk factors for chronic liver disease is incorrect, although this may
influence initiation of anti-viral therapy per se.

C Fibrosis on liver biopsy


Fibrosis on liver biopsy is incorrect, although this too is likely to drive a decision to
initiate anti-viral therapy.

E Transaminases

Transaminases is incorrect. These are a poor indicator of the degree of underlying fibrosis.
40972

Rate this question:

Next Question

Previous Question Tag Question

Feedback End Review

Difficulty: Average

Peer Responses %

Q. Answered Flagged

Q1

Q2

Q3

Q4

Q5

Q6

Q7
Q. Answered Flagged

Q8
0:00:07/03:00:00

A 51-year-old man who has a history of smoking and type 2 diabetes controlled with
metformin monotherapy comes to the Emergency Department with 30 min of central
crushing chest pain radiating to both arms. Examination reveals a BP of 105/70 mmHg and
a pulse of 95 bpm and regular. There are bilateral basal crackles on auscultation of his
chest, although he is able to lie flat.

Investigations:

Hb 13.1 g/dl

WCC 10.1 x 109/l

PLT 203 x 109/l

Na+ 137 mmol/l

K+ 4.5 mmol/l

Creatinine 104 micromol/l

Glucose 18.2 mmol/l

ECG Anterior ST elevation >2mm

CXR No mediastinal widening

A Aspirin, clopidogrel, low-molecular weight (LMW) heparin

B Aspirin, clopidogrel, fondaparinux

C Aspirin, clopidogrel, unfractionated heparin

D Aspirin, clopidogrel, bivalirudin

E Aspirin, ticagrelor, abciximab

Explanation 
D Aspirin, clopidogrel, bivalirudin

This trio of medications is now the recommendation for patients whilst awaiting primary PCI
(2013 NICE guidelines on acute myocardial infarction (MI)). If PCI cannot be performed
within 120 min of a diagnosis of ST elevation myocardial infarction (STEMI) being made, then
thrombolysis is recommended as an alternative.

A Aspirin, clopidogrel, low-molecular weight (LMW) heparin

Aspirin, clopidogrel, low-molecular weight (LMW) heparin is incorrect. This trio of


treatment is for non-ST elevation myocardial infarction (NSTEMI) and not for STEMI.

B Aspirin, clopidogrel, fondaparinux

Aspirin, clopidogrel, fondaparinux is incorrect. Fondaparinux is the anticoagulant of choice in


patients diagnosed with NSTEMI, not STEMI.

C Aspirin, clopidogrel, unfractionated heparin

Aspirin, clopidogrel, unfractionated heparin is incorrect. Unfractionated heparin is rarely


used in treatment of MI, with many newer agents available.

E Aspirin, ticagrelor, abciximab

Aspirin, ticagrelor, abciximab is incorrect. Ticagrelor can be used as an alternative anti-


platelet option post STEMI and then continued for up to 12 months. IIbIIIa inhibitors are not
recommended as part of routine therapy for patients with STEMI.
37787

Rate this question:

Next Question

Previous Question Tag Question

Feedback End Review

Difficulty: Difficult

Peer Responses %
Q. Answered Flagged

Q1

Q2

Q3

Q4

Q5

Q6

Q7

Q8

Q9

nice.org.uk/guidance/cg167
(https://www.nice.org.uk/guidance/cg167)
0:00:07/03:00:00

A surgeon is presenting data on a new form of stent used in the treatment of acute coronary
syndrome. The protocol is complex with respect to anti-coagulation and management of
blood pressure, and there were some protocol violators in the trial.

He intends to present an intention to treat analysis.

A All patients who presented for screening

B All patients allocated to each treatment group

C Excludes patients lost to follow up

D Only those patients treated according to the protocol

E Only those patients who completed the study

Explanation 

B All patients allocated to each treatment group

In this situation an intention to treat analysis, which includes all patients allocated to each
treatment group is the most appropriate way to present the data. This is because side-effects
of the therapy or differences in efficacy may drive changes in treatment behaviour after
randomisation, thus meaning that only presenting patients treated per protocol or
completers, risks giving a picture of clinical efficacy which is not reflective of use in the clinic.

A All patients who presented for screening

All patients who presented for screening is incorrect. Screen failures are not included in the
analysis, although regulators are increasingly asking for information on screen failures, as
these may indicated potential off license use of a treatment after launch.

C Excludes patients lost to follow up

Excludes patients lost to follow up is incorrect. All data on patients lost to follow up is
included in the intention to treat analysis until the point where no further data is available.
D Only those patients treated according to the protocol

Only those patients treated according to the protocol is incorrect. This is not an ITT, but a
per protocol analysis.

E Only those patients who completed the study

Only those patients who completed the study is incorrect; this would be a “completer”
analysis. It risks censoring data on treatment failure or on adverse events of therapy, and is
therefore not optimal.
40983

Rate this question:

Next Question

Previous Question Tag Question

Feedback End Review

Difficulty: Average

Peer Responses %

Q. Answered Flagged

Q1

Q2

Q3

Q4

Q5
Q. Answered Flagged

Q6

Q7


Q8
0:00:07/03:00:00

A young pregnant woman would like to know the risk of having a child with genetic
abnormalities. Her two brothers are affected by haemophilia, which is inherited as an X-linked
disease. She is completely asymptomatic and has not received genetic screening. She knows
from her last ultrasound test that her baby is a boy.

A 1 in 2

B 1 in 4

C 1 in 6

D 1 in 8

E 1 in 16

Explanation 

B 1 in 4

We do not know this woman’s status with respect to haemophilia gene carriage, but given
two of her siblings are affected, her chance of carrying an abnormal X chromosome is 1 in 2. If
she has an affected gene, her chance of passing it on is 1 in 2, giving an overall chance of
passing on haemophilia to a son of 1 in 4.

A 1 in 2

1 in 2 is incorrect. 1 in 2 is the chance of a male child inheriting the disease if the pregnant
woman is a proven carrier.

C 1 in 6

1 in 6 is incorrect. 1 in 6 is too low to reflect the risk of inheritance for an X-linked disorder.
This reflects the chance of a child inheriting an autosomal recessive condition if one parent
is a confirmed carrier and the other is from a family with siblings who have a recessive
disease but has not been confirmed as a carrier or a sufferer themselves.
D 1 in 8

1 in 8 is incorrect. 1 in 8 is too low to represent the rate of inheritance of an autosomal


recessive disease where both parents are confirmed as carriers.

E 1 in 16

1 in 16 is incorrect. Even the most common autosomal recessive disorder would not carry this
inheritance rate, 1 in 16, where one patient was a carrier and the other had unknown status.
48690

Rate this question:

Next Question

Previous Question Tag Question

Feedback End Review

Difficulty: Average

Peer Responses %

Q. Answered Flagged

Q1

Q2

Q3

Q4

Q5

Q6
Q. Answered Flagged

Q7

Q8

0:00:07/03:00:00

A 19-year-old male escort, who is known to be HIV positive comes to the Emergency
Department for review. He has severe pain and mucopurulent, bloody, anal discharge over the
past 48hrs. He admits to unprotected intercourse with a number of males at a party after
taking crystal methamphetamine. His temperature is 37.8°C, pulse is 80/min and regular, and
BP is 105/80 mmHg. You can see pus and blood on his underwear, and he is unable to
tolerate rectal examination due to pain.

A Azithromycin 2g PO

B Cefixime 400mg PO

C Ceftriaxone 500mg IM and Azithromycin 1g PO

D Doxycycline 100mg BD for 1 week

E Olfloxacin 400mg PO

Explanation 

C Ceftriaxone 500mg IM and Azithromycin 1g PO

This combination is the recommended first line intervention for the treatment of
gonorrhoea, and has the advantage that it can be given whilst the patient is in the clinic
after the diagnosis has been confirmed. The presence of anal pain and discharge post
unprotected intercourse fits well with gonococcal proctitis as the underlying cause.
Screening for other sexually transmitted infections is of course essential.

A Azithromycin 2g PO

Azithromycin 2g is incorrect, although this is an alternative in patients who refuse IM


therapy. It is often poorly tolerated due to its GI adverse event profile.

B Cefixime 400mg PO
Cefixime 400mg is incorrect, although this is the preferred alternative for patients who refuse
IM therapy.

D Doxycycline 100mg BD for 1 week

Doxycycline 100mg BD is incorrect. Tetracyclines are usually avoided because of potential


resistance.

E Olfloxacin 400mg PO

Olfloxacin 400mg PO is incorrect. This is rarely used as a single agent because of potential
resistance.
41092

Rate this question:

Next Question

Previous Question Tag Question

Feedback End Review

Difficulty: Average

Peer Responses %

Q. Answered Flagged

Q1

Q2

Q3

Q4
Q. Answered Flagged

Q5

Q6

Q7

Q8 
0:00:07/03:00:00

You are asked to audit the performance with respect to management of acute coronary
syndrome versus a recent change in hospital guidelines to introduce use of Ticagrelor.

A COX 1 inhibitor

B COX 2 inhibitor

C Direct thrombin inhibitor

D Glycoprotein 2b3a inhibitor

E Platelet ADP receptor inhibitor

Explanation 

E Platelet ADP receptor inhibitor

Ticagrelor blocks adenosine diphosphate (ADP) receptors of subtype P2Y12. In contrast to


the other antiplatelet drugs such as Clopidogrel and Ticlodipine, Ticagrelor has a binding
site different from ADP, making it an allosteric antagonist, and the blockade is reversible. It
does not undergo hepatic activation making it less vulnerable to p450 interaction or
polymorphisms vs other agents such as Clopidogrel.

A COX 1 inhibitor

COX 1 inhibitor is incorrect. COX 1 inhibitors inhibit the production of thromboxane; aspirin is
a prime example.

B COX 2 inhibitor

COX 2 inhibitor is incorrect. These were developed as “safe” anti-inflammatories but were
found to cause myocardial infarction. This has now severely limited their use.

C Direct thrombin inhibitor


Direct thrombin inhibitor is incorrect. Lepirudin is an example of a direct thrombin inhibitor.

D Glycoprotein 2b3a inhibitor

Glycoprotein 2b3a inhibitor is incorrect. Abciximab is an example of a platelet 2b3a receptor


inhibitor, the class inhibits platelet aggregation and thrombus formation.
41159

Rate this question:

Next Question

Previous Question Tag Question

Feedback End Review

Difficulty: Average

Peer Responses %

Q. Answered Flagged

Q1

Q2

Q3

Q4

Q5

Q6

Q7

Q8
Q. Answered Flagged 
0:00:07/03:00:00

A 28-year-old man who is known to be HIV-positive presents to the Emergency Department


with gradually increasing drowsiness, and problems moving his left arm. His partner tells you
he has become increasingly confused over the past 48 hours and has been complaining of
severe headaches. His GCS is 13 and there is obvious meningism. Fundoscopy reveals
bilateral optic disc swelling. MRI scanning reveals multiple ring enhancing lesions and
serology for toxoplasma is positive.

A Atovaquone and Pyrimethamine and folinic acid

B Atovaquone and Sulfadiazine

C Azithromycin and Pyrimethamine and folinic acid

D Clindamycin and Pyrimethamine

E Pyrimethamine and Sulfadiazine and folinic acid

Explanation 

E Pyrimethamine and Sulfadiazine and folinic acid

This regime is more effective than Atovaquone or Clindamycin containing regimens, and is
therefore the preferred initial therapy. It is however associated with cutaneous
hypersensitivity, and if this occurs, an Atovaquone or Clindamycin based regime may be
required. Corticosteroids are also recommended for patients with evidence of raised
intracranial pressure.

A Atovaquone and Pyrimethamine and folinic acid

Atovaquone and Pyrimethamine and folinic acid is incorrect. This is a second line regimen
where switching is required because of tolerability issues.

B Atovaquone and Sulfadiazine


Atovaquone and Sulfadiazine is incorrect. This too is a second line regimen where switching
is necessary because of tolerability.

C Azithromycin and Pyrimethamine and folinic acid

Azithromycin and Pyrimethamine and folinic acid is incorrect. This is an alternative regimen
for patients unable to tolerate sulfa containing compounds.

D Clindamycin and Pyrimethamine

Clindamycin and Pyrimethamine is incorrect. Clindamycin and Pyrimethamine is the generally


preferred second line regimen for patients unable to take sulfa- containing compounds.
40987

Rate this question:

Next Question

Previous Question Tag Question

Feedback End Review

Difficulty: Average

Peer Responses %

Q. Answered Flagged

Q1

Q2

Q3

Q4
Q. Answered Flagged

Q5

Q6

Q7

Q8 
0:00:07/03:00:00

A 34-year-old postgraduate student from East Africa comes to the Emergency Department
for review. He has become increasingly lethargic over the past 10 days with drowsiness,
apathy and low-grade fever. Over the past 48hrs he has begun to suffer from an
increasingly severe headache and neck stiffness and now has diplopia. On examination he
is pyrexial 38.2°C, pulse is 88/min and regular. He has meningism, a left VI nerve palsy and
blurring of the margins of the left optic disc.

Investigations:

Hb 11.4 g/dl

WCC 12.3 x109/l

PLT 155 x109/l

Na+ 132 mmol/l

K+ 4.3 mmol/l

Creatinine 110 micromol/l

CRP 201 mg/l

CT head no focal lesion

Lumbar puncture elevated protein, decreased glucose, mononuclear pleocytosis

Quantiferon gamma positive

HIV negative

A 2 weeks

B 3 months

C 6 months

D 9 months

E 12 months
Explanation 

E 12 months

In general a four-drug regimen is recommended for the first two months of therapy in TB
meningitis, followed by the combination of Isoniazid and Rifampicin for a further 10
months. Corticosteroids are also recommended in patients who are HIV negative for the
first 2-3 weeks after diagnosis. Where there is a tuberculoma, therapy for a more
prolonged period (up to 18 months) should be considered.

A 2 weeks

2 weeks is incorrect. This is the typical duration of therapy for a patient with non-tuberculous
bacterial meningitis.

B 3 months

3 months is incorrect. 3 months of dual therapy is the typical duration of therapy for latent
tuberculosis.

C 6 months

6 months is incorrect. 6 months of therapy is the typical duration of therapy for a patient
with active respiratory TB.

D 9 months

9 months is incorrect. 9 months of therapy runs the risk of inadequate clearance of CNS TB.
40989

Rate this question:

Next Question

Previous Question Tag Question

Feedback End Review

Difficulty: Average

Peer Responses %
Q. Answered Flagged

Q1

Q2

Q3

Q4

Q5

Q6

Q7

Q8

Q9

0:00:07/03:00:00

A 45-year-old woman comes to the Neurology Clinic for review. She complains of severe
stabbing pains lasting from a few seconds to up to 2 minutes, always affecting the upper part
of the left hand side of her face, triggered by cleaning her teeth, eating cold foods, or wind
on a cold winter’s day. The pain was not improved by non-steroidals but has improved after
her GP prescribed Carbamazepine. Examination reveals a normal blood pressure, and a
completely normal neurological examination.

A Cluster headache

B Migraine

C Paroxysmal hemicrania

D Short-lasting Unilateral Neuralgiform headache attacks with Conjunctival


injection and Tearing

E Trigeminal neuralgia

Explanation 

E Trigeminal neuralgia

The age and sex at presentation, coupled with the duration of headaches and triggers such
as cleaning teeth or cold wind, fit well with a diagnosis of trigeminal neuralgia. The
condition does not usually respond to conventional analgesics such as NSAIDs or
Paracetamol, but does respond to Carbamazepine, again supportive of the diagnosis.
Lamotrigine and Baclofen are other agents that have been trialled for the condition, and
surgery is of value in patients who fail to gain pain control on medical therapy alone.

A Cluster headache

Cluster headache is incorrect, given the absence of somatic symptoms such as tearing or
conjunctival injection, and the presence of triggers.

B Migraine
Migraine is incorrect. Migraine is typically not unilateral, and is preceded by an aura.

C Paroxysmal hemicrania

Paroxysmal hemicrania is incorrect. Symptoms are similar to cluster headache, and there is
often a response to NSAIDs.

D Short-lasting Unilateral Neuralgiform headache attacks with Conjunctival


injection and Tearing

SUNCT is incorrect. SUNCT stands for Short-lasting unilateral neuralgiform headache attacks
with conjunctival injection and tearing, of which the somatic symptoms are clearly absent
here.
41100

Rate this question:

Next Question

Previous Question Tag Question

Feedback End Review

Difficulty: Average

Peer Responses %

Q. Answered Flagged

Q1

Q2

Q3
Q. Answered Flagged

Q4

Q5

Q6

Q7

0:00:07/03:00:00

A 70-year-old woman is admitted to the Emergency Department having taken overdose of


Diazepam with wine after having a row with her daughter. She sent a number of goodbye
texts, left a suicide note and timed her overdose for when she thought no one would be
visiting the house. She lives alone and has no partner, and has taken a previous overdose 1
year earlier. Past medical history of hypertension is noted.

A Age

B Female sex

C History of hypertension

D Living alone

E Previous overdose

Explanation 

E Previous overdose

The strongest factor predictive of suicide is prior history of attempted suicide. Those with a
prior history of suicide attempts are up to 6 times more likely to make another attempt.
One of every 100 suicide-attempt survivors will die by suicide within one year of their index
attempt, a risk approximately 100 times that of the general population.

A Age

Age is incorrect. Suicide is commonest in young males, and in men >85 years.

B Female sex

Female sex is incorrect. Male sex is associated with greater risk of successful suicide.

C History of hypertension
History of hypertension is incorrect. Hypertension is not necessarily associated with physical
impairment; it is not therefore a significant factor here.

D Living alone

History of living alone is incorrect. Living alone is associated with increased risk of successful
suicide, although the contribution to risk is less than that of previous suicide attempt.
41059

Rate this question:

Next Question

Previous Question Tag Question

Feedback End Review

Difficulty: Average

Peer Responses %

Q. Answered Flagged

Q1

Q2

Q3

Q4

Q5

Q6

Q7
Q. Answered Flagged

Q8
0:00:07/03:00:00

An obese 28-year-old woman presents to the clinic for review, complaining of increased
shortness of breath over the past 4-6 months. She has gone from being able to fast-walk to
catch a train, to only being able to walk a few metres without needing to stop. She has no
past medical history of note, smokes 5 cigarettes per day and drinks no alcohol. On
examination her BP is 155/95 mmHg; pulse is 78/min and regular. Her BMI is 33. Chest is
clear, although air entry is diminished at both bases. There is no wheeze or crepitation. O2
saturations are 94% on air.

Investigations:

Hb 13.1 g/dl

WCC 8.2 x109/l

PLT 201 x109/l

Na+ 137 mmol/l

K+ 4.5 mmol/l

Bicarbonate 32 mmol/l

Creatinine 103 micromol/l

PaO2 9.9 kPa

PaCO2 6.8 kPa

A Asthma

B COPD

C Idiopathic pulmonary fibrosis

D Obesity hypoventilation syndrome

E Thromboembolic disease

Explanation 
D Obesity hypoventilation syndrome

This patient has alveolar hypoventilation while awake, as exhibited by the raised CO2, and
in the absence of obvious other underlying chest pathology, obesity hypoventilation
syndrome is the most likely cause. The condition often co-exists with obstructive sleep
apnoea (OSA), and it is important to question this patient about symptoms of daytime
sleepiness. Weight loss is the mainstay of therapy, with the addition of nocturnal CPAP if
OSA is confirmed.

A Asthma

Asthma is incorrect. The absence of symptoms of wheeze and cough, and a clear chest on
auscultation count against a diagnosis of asthma.

B COPD

COPD is incorrect. This patient only has a minor history of smoking and absence of signs on
auscultation, counting against COPD as a diagnosis.

C Idiopathic pulmonary fibrosis

Idiopathic pulmonary fibrosis is incorrect. In this situation obesity hypoventilation syndrome


is much more common, and the absence of signs of pulmonary fibrosis counts against the
diagnosis.

E Thromboembolic disease

Thromboembolic disease is incorrect. The absence of symptoms of chest pain counts against
a pulmonary embolus. In addition the more usual ABG picture is one of hypoxia and
hypocapnia because of respiratory drive.
41076

Rate this question:

Next Question

Previous Question Tag Question

Feedback End Review

Difficulty: Average

Peer Responses %
Q. Answered Flagged

Q1

Q2

Q3

Q4

Q5

Q6

Q7

Q8

Q9

0:00:07/03:00:00

A 62-year-old man presents to the clinic with symptoms of headaches and ataxia, which have
increased over the past 6 months. He has a past history of hypertension and an inferior MI
some 10 years earlier, but is otherwise well. There is no history of tinnitus or hearing loss.
Examination reveals a BP of 135/82 mmHg; pulse is 75/min and regular. There is right-
sided ataxia. Routine bloods are unremarkable, MRI scanning reveals a right
cerebellopontine angle (CPA) tumour, which enhances with contrast, and has a
heterogeneous appearance including areas of calcification.

A Arachnoid cyst

B Epidermoid tumour

C Glioblastoma

D Meningioma

E Vestibular schwannoma

Explanation 

D Meningioma

The lack of tinnitus or hearing loss counts against a vestibular schwannoma being the
underlying cause of the symptoms seen here. Hearing loss is a late presenting feature in
patients with a CPA related to an underlying meningioma and therefore fits with the clinical
picture. MRI showing areas of calcification further supports the diagnosis of meningioma;
enhancement of the dura adjacent to the tumour may also been seen on the scan.

A Arachnoid cyst

Arachnoid cyst is incorrect. Arachnoid cysts usually appear as a hypointense, homogenous


mass on MRI.

B Epidermoid tumour
Epidermoid tumour is incorrect. Epidermoid tumours are non-enhancing and usually
present at an earlier age.

C Glioblastoma

Glioblastoma is incorrect. Glioblastomas are rapidly growing and present with significant
neurological symptoms including seizures, memory loss and decreased consciousness due to
pressure from the tumour and surrounding oedema.

E Vestibular schwannoma

Vestibular schwannoma is incorrect. The lack of hearing loss or tinnitus does not fit with a
vestibular schwannoma.
40968

Rate this question:

Next Question

Previous Question Tag Question

Feedback End Review

Difficulty: Average

Peer Responses %

Q. Answered Flagged

Q1

Q2

Q3
Q. Answered Flagged

Q4

Q5

Q6

Q7

0:00:07/03:00:00

What is the main feature of DNA sequence polymorphisms that differentiates them from
mutations?

A They are common in the population

B They are evenly spread throughout all genes

C They are not found in certain ethnic groups

D They often cause serious medical conditions

E They usually interfere with normal gene function

Explanation 

A They are common in the population

A polymorphism is a DNA sequence variant that is common in the population, whereas a


mutation is a change in a DNA sequence away from normal that results in an abnormal rare
variant; however, these definitions are not absolute:

The usual cut-off point between a mutation and a polymorphism is 1%


Therefore, if the DNA sequence variant has a frequency of 1% or higher, it is classed
as a polymorphism, whereas it is regarded as a mutation if the frequency is lower
than 1%

B They are evenly spread throughout all genes

They are evenly spread throughout all genes is incorrect. DNA sequence polymorphisms are
not evenly spread through all genes. Some genes are more polymorphic than others.

C They are not found in certain ethnic groups

They are not found in certain ethnic groups is incorrect. Polymorphic sequence variants are
found in all populations, not just in certain ethnic groups.
D They often cause serious medical conditions

They often cause serious medical conditions is incorrect. They usually do not cause clinically
serious diseases. However, polymorphic sequence variation does affect disease susceptibility
and can also influence drug responses.

E They usually interfere with normal gene function

They usually interfere with normal gene function is incorrect. DNA sequence
polymorphisms do not usually interfere with normal gene function. Many polymorphisms
are found outside genes and have no effect on gene function; others may be found within
genes and modify gene function to influence characteristics such as height and hair colour
rather than being pathogenic
48104

Rate this question:

Next Question

Previous Question Tag Question

Feedback End Review

Difficulty: Average

Peer Responses %

Q. Answered Flagged

Q1

Q2

Q3
Q. Answered Flagged

Q4

Q5

Q6

Q7

0:00:07/03:00:00

A 40-year-old woman with severe rheumatoid arthritis is referred to you from the
rheumatologists. She has been short of breath over the last 2 months and her chest X-ray
shows some widespread patchy shadowing. She has bilateral inspiratory crepitations and a
low-grade fever of 37.8°C. She has been taking methotrexate for her arthritis for 4 years.
You perform a computed tomography scan, which shows multifocal consolidation.

A Methotrexate pneumonitis

B Organising pneumonia

C Pneumocystis pneumonia

D Rheumatoid lung fibrosis

E Tuberculosis

Explanation 

B Organising pneumonia

Organising pneumonia can occur in rheumatoid arthritis, with fever, dyspnoea and multifocal
consolidation. This responds dramatically to steroids.

A Methotrexate pneumonitis

Methotrexate pneumonitis is incorrect. Methotrexate-induced pneumonitis usually occurs


within 4 months of starting the drug – a CT scan would show pulmonary infiltrates.

C Pneumocystis pneumonia

Pneumocystis pneumonia is incorrect. Opportunistic pulmonary infection resulting from the


use of immunosuppressive drugs can occur, but Pneumocystis pneumonia would not give
these computed tomographic (CT) appearances.

D Rheumatoid lung fibrosis


Rheumatoid lung fibrosis is incorrect. Fibrosis is found in up to 60% of patients with
rheumatoid arthritis, but chest X-ray changes are seen in only 5%. Symptomatic disease is
unusual, although patients can present with a dry cough and dyspnoea, and fine crackles
might be heard on auscultation.

E Tuberculosis

Tuberculosis is incorrect. Opportunistic pulmonary infection resulting from the use of


immunosuppressive drugs can occur, but tuberculosis would not give these CT appearances.
45246

Rate this question:

Next Question

Previous Question Tag Question

Feedback End Review

Difficulty: Difficult

Peer Responses %

Q. Answered Flagged

Q1

Q2

Q3

Q4

Q5

Q6
Q. Answered Flagged

Q7

Q8

0:00:07/03:00:00

A 56-year-old man with ischaemic heart disease and COPD comes to the Emergency
Department for review, complaining of generalised muscle aches and pains and severe
lethargy. Apparently he attended the GP around a week earlier with a chronic cough and
was prescribed antibiotics. At the same time he was given a general review of his
medication. Pre- existing therapies include Lisinopril, Aspirin, Simvastatin and Ranitidine. His
BP is 132/80 mmHg, pulse is 85/min and regular, he is apyrexial. Auscultation of the chest
reveals bilateral wheeze, but no focal signs. There is minor tenderness in both calves.

Investigations:

Hb 13.1 g/dl

WCC 8.2 x109/l

PLT 184 x109/l

Na+ 137 mmol/l

K+ 5.6 mmol/l

Creatinine 115 micromol/l

CK 2815 U/l

ECG SR, old inferior MI

A Ciprofloxacin

B Clarithromycin

C Clopidogrel

D Omeprazole

E Prednisolone

Explanation 

B Clarithromycin
Macrolide antibiotics are well recognised as inhibitors of CYP 3A4. The presentation with
muscle aches and pains and marked elevation in CK is consistent with excess Simvastatin
exposure as a result of inhibition of its metabolism, and Clarithromycin is the most likely
cause. Other important inhibitors of 3A4 in this context include: azole anti-fungals, Diltiazem,
Verapamil, and of course, grapefruit juice.

A Ciprofloxacin

Ciprofloxacin is incorrect. Ciprofloxacin is primarily an inhibitor of CYP 1A2.

C Clopidogrel

Clopidogrel is incorrect. Clopidogrel is an inhibitor of CYP 2B6.

D Omeprazole

Omeprazole is incorrect. Omeprazole is an inhibitor of CYP 2C19.

E Prednisolone

Prednisolone is incorrect. Prednisolone is an inducer of CYP 3A4, in all likelihood leading to a


reduction in the effects of Simvastatin, rather than enhancement.
41020

Rate this question:

Next Question

Previous Question Tag Question

Feedback End Review

Difficulty: Average

Peer Responses %
Q. Answered Flagged

Q1

Q2

Q3

Q4

Q5

Q6

Q7

Q8

Q9

0:00:07/03:00:00

A 61-year-old man who is known to drink 6-8 pints of beer per day comes to the Neurology
Clinic for review. He says problems began in his lower legs about a year ago, with loss of
vibration and position sense in both legs. Over time this has progressed to weakness with
increased stiffness, and some juddering movements affecting both legs. He uses two sticks to
walk only a few metres and has arrived at the clinic in a wheelchair. He admits to urinary
incontinence. On examination his BP is 130/80 mmHg sitting, he has to be helped to stand,
but this is associated with a 25mmHg drop in systolic BP. Neurological exam confirms distal
loss of fine touch and proprioception, bilateral increased tone, and diminished knee jerk
reflexes.

A Vitamin A

B Vitamin B1

C Vitamin B6

D Vitamin B12

E Vitamin E

Explanation 

D Vitamin B12

The neurological picture here is consistent with subacute combined degeneration of the cord,
as a result of B12 deficiency. Blood testing is likely to reveal macrocytic anaemia, and B12
levels will confirm deficiency. Where B12 deficiency has led to neurological deficit,
hydroxycobalamin is usually given IM on alternate days until neurological symptom
improvement has reached a plateau; it is then given at 1mg every 2 months.

A Vitamin A

Vitamin A is incorrect. Vitamin A deficiency initially leads to night blindness, later on this can
lead to xerophthalmia and keratomalacia.
B Vitamin B1

Vitamin B1 is incorrect. Thiamine deficiency leads to Wernicke’s encephalopathy.

C Vitamin B6

Vitamin B6 is incorrect. Initially vitamin B6 deficiency results in a seborrhoeic dermatitis type


picture.

E Vitamin E

Vitamin E is incorrect. Vitamin E deficiency results in spinocerebellar ataxia, myopathy and


dysarthria.
41155

Rate this question:

Next Question

Previous Question Tag Question

Feedback End Review

Difficulty: Average

Peer Responses %

Q. Answered Flagged

Q1

Q2

Q3
Q. Answered Flagged

Q4

Q5

Q6

Q7

Q8 
0:00:07/03:00:00

A 44-year-old man of Han Chinese descent is reviewed on the Neurology Ward after
admission with complex partial seizures. A decision is made to start him on Carbamazepine
therapy, but you are aware that this agent is associated with increased risk of Stevens
Johnson syndrome.

A Acetyltransferase activity

B Aromatase activity

C Cytochrome p450 panel

D HLA type

E Sulphatase activity

Explanation 

D HLA type

HLA typing is recognised to be increasingly useful in predicting the risk of Stevens Johnson
syndrome in patients prescribed Carbamazepine. HLA-B*1502 screening is now
recommended for all patients of Han Chinese or Thai origin before commencing therapy. In
European populations the HLA-A*3101 type is recognised to be associated with increased
risk of Stevens Johnson.

A Acetyltransferase activity

Acetyltransferase activity is incorrect. N-acetyl transferase polymorphisms are recognised to


be associated with risk of drug reactions in patients prescribed hydralazine-type drugs.

B Aromatase activity

Aromatase activity is incorrect. Aromatase activity is related to peripheral conversion of sex


steroids.
C Cytochrome p450 panel

Cytochrome p450 panel is incorrect. Carbamazepine is metabolised via the 3A4 p450
enzyme, interactions are therefore more important than polymorphisms.

E Sulphatase activity

Sulphatase activity is incorrect. Sulphatase is responsible for metabolism of steroid


hormones.
41099

Rate this question:

Next Question

Previous Question Tag Question

Feedback End Review

Difficulty: Average

Peer Responses %

Q. Answered Flagged

Q1

Q2

Q3

Q4

Q5

Q6
Q. Answered Flagged

Q7

Q8

medicines.org.uk/emc/medicine/1329#CLINICAL_PRECAUTIONS
(http://www.medicines.org.uk/emc/medicine/1329#CLINICAL_PRECAUTIONS)
0:00:07/03:00:00

A 42-year-old man who sleeps rough and is well known to the Emergency Department
because of alcoholism, is brought to the Department after a collapse. According to fellow
rough sleepers he has been unwell with fevers and a cough for a few days and had not
moved from his spot for the past 12 hours. There are multiple previous attendances for
intoxication; he has been known to use IV heroin and crack cocaine in the past, although
screening for HIV has proved negative within the past 3 months.

On examination, he is pyrexial 38.8°C, pulse is 95/min and regular, and his BP is 95/60
mmHg. There are extensive bilateral coarse crackles and wheeze on auscultation. He has a
quiet pansystolic murmur. There are signs of chronic liver disease and he looks emaciated.

Investigations:

Hb 10.0 g/dl

WCC 15.2 x109/l

PLT 85 x109/l

CRP 189 mg/l

Na+ 133 mmol/l

K+ 3.5 mmol/l

Creatinine 95 micromol/l

ALT 120 U/l

ALP 174 U/l

CXR patchy shadowing bilaterally, multiple areas of possible cavitation

A Klebsiella pneumoniae

B Mycoplasma pneumoniae

C Pneumocystis jirovecii

D Staphylococcus aureus

E Streptococcus pneumoniae
Explanation 

A Klebsiella pneumoniae

KP is particularly prevalent in patients who are immunocompromised and/ or suffering from


alcoholism. They present with symptoms of community-acquired pneumonia, often with signs
of underlying chronic liver disease, and CXR reveals evidence of consolidation with cavitation.
Third generation cephalosporins and quinolones have a high level of activity against
Klebsiella, although sensitivity testing is essential.

B Mycoplasma pneumoniae

Mycoplasma pneumoniae is incorrect. Mycoplasma pneumoniae is not usually associated with


cavitation.

C Pneumocystis jirovecii

Pneumocystis jirovecii is incorrect. It is associated with patchy consolidation, but not usually
with cavitation. Also of note is the fact that he has at least recently tested HIV negative.

D Staphylococcus aureus

Staphylococcus aureus is incorrect, although it would be an alternative diagnosis if testing for


Klebsiella proved negative. The alcoholism drives us towards Klebsiella as the first choice
diagnosis here.

E Streptococcus pneumoniae

Streptococcus pneumoniae is incorrect. It is not usually associated with cavitating


pneumonia.
41094

Rate this question:

Next Question

Previous Question Tag Question

Feedback End Review

Difficulty: Average
Peer Responses %

Q. Answered Flagged

Q1

Q2

Q3

Q4

Q5

Q6

Q7

Q8

Q9

0:00:07/03:00:00

A 50-year-old man presents to his GP with multiple white umbilicated lumps on his eyelids.
He is HIV-positive.

A Multiple basal cell carcinomas

B Chalazia

C Molluscum contagiosum

D Kaposi's sarcoma

E Seborrhoeic keratosis

Explanation 

C Molluscum contagiosum

Molluscum contagiosum is caused by the poxvirus. These are raised white, pink or flesh-
coloured, umbilicated and may occur as a single or multiple lesions. They are usually seen in
children or immunocompromised adults.

A Multiple basal cell carcinomas

Mutiple basal cell carcinomas is incorrect. Multiple basal cell carcinomas may occur and
should be part of the differential diagnosis, but they are not commonly associated with HIV.
They typically have a rolled edge with central ulceration, with a pearly appearance.

B Chalazia

Chalazia is incorrect. Chalazia occur as an inflammatory response to Meibomian gland


dysfunction. These may appear as raised, flesh-coloured lesions in the eyelid, but would not
have an umbilicated appearance. They are common in the immunocompetent.

D Kaposi's sarcoma
Kaposi’s sarcoma is incorrect. While this is classically associated with acquired
immunodeficiency syndrome (AIDS), this presents as a purplish-red nodule or macule
without central umbilication. It is caused by human herpes virus 8 (HHV-8).

E Seborrhoeic keratosis

Seborrhoeic keratosis is incorrect. This occurs in middle-aged to elderly adults as


pedunculated lesions or plaques. This is a benign occurrence associated with ageing, and is
extremely common in the immunocompetent.
50706

Rate this question:

Next Question

Previous Question Tag Question

Feedback End Review

Difficulty: Easy

Peer Responses %

Q. Answered Flagged

Q1

Q2

Q3

Q4

Q5

Q6
Q. Answered Flagged

Q7

Q8
0:00:07/03:00:00

A 41-year-old business man presents to the Emergency Department suffering from bloody
diarrhoea of one day’s duration. He returned from a conference some 48hrs earlier, and
overnight has developed a fever, abdominal cramps, and opened his bowels some 10 times.
There is no past medical history of note and he tells you that two of his colleagues have
been ill with similar symptoms. On examination he is pyrexial 38.8°C, BP is 122/80 mmHg;
pulse is 95/min and regular. He is tender in the right iliac fossa on abdominal palpation.
Stool PCR for salmonella is negative.

A Campylobacter

B Norovirus

C Shigella

D Staphylococcus

E Typhoid fever

Explanation 

A Campylobacter

The period of approximately 2 days between possible exposure and the onset of symptoms
of abdominal pain and bloody diarrhoea fits well with a diagnosis of Campylobacter. Mass
catering, particularly where poultry products may have not been adequately cooked,
presents a significant risk of infection. Multiple faecal samples should be sent for analysis, as
a single sample may well be negative on culture. Patients should be excluded from school or
work until 48hrs after the last episode of diarrhoea. Antibiotic use is controversial, but can
be considered if symptoms are particularly severe, (although NICE dues not support their
use).

B Norovirus

Norovirus is incorrect. Norovirus results in severe nausea, vomiting and diarrhoea, but usually
follows only a 24hr course. Bloody diarrhoea usually is not seen.
C Shigella

Shigella is incorrect. Shigella is associated with outbreaks of dysentery, and is most


commonly associated with children’s day care establishments.

D Staphylococcus

Staphylococcus is incorrect. Staphylococcus food poisoning is associated with a toxin usually


found in dairy products and presents with symptoms only a few hours after ingestion.

E Typhoid fever

Typhoid fever is incorrect. Patients with typhoid fever tend to have constipation rather than
diarrhoea.
40974

Rate this question:

Next Question

Previous Question Tag Question

Feedback End Review

Difficulty: Average

Peer Responses %

Q. Answered Flagged

Q1

Q2
Q. Answered Flagged

Q3

Q4

Q5

Q6

Q7

0:00:07/03:00:00

A 28-year-old woman is admitted with an episode of severe depression some 6 months after
the birth of her first child. She attends the Emergency Department with her husband, who
works as a neurology researcher and is concerned that some of the drugs she is prescribed
may cause galactorrhoea.

A Amisulpride

B Chlorpromazine

C Olanzapine

D Risperidone

E Sertraline

Explanation 

E Sertraline

Although SSRIs such as Sertraline do cause galatorrhoea because of raised prolactin, it is


classified as rare in the case of Sertraline, meaning it occurs at a frequency of between 1 in
1000 and 1 in 10000. Hence out of the options given it would be seen as least likely to cause
galactorrhoea.

A Amisulpride

Amisulpride is incorrect. Elevated prolactin is listed as a common finding in patients


prescribed Amisulpride; in a significant number of patients this becomes symptomatic,
leading to galactorrhoea.

B Chlorpromazine

Chlorpromazine is incorrect. Chlorpromazine is commonly associated with elevated prolactin;


hence galactorrhoea is potentially seen much more frequently versus Sertraline.
C Olanzapine

Olanzapine is incorrect. Olanzapine, being an atypical anti-psychotic is less likely to lead to


symptomatic hyperprolactinaemia, although some degree of elevation is seen in up to half of
patients prescribed it.

D Risperidone

Risperidone is incorrect. 90% of patients prescribed Risperidone see at least some elevation
in their prolactin level.
41156

Rate this question:

Next Question

Previous Question Tag Question

Feedback End Review

Difficulty: Average

Peer Responses %

Q. Answered Flagged

Q1

Q2

Q3

Q4

Q5
Q. Answered Flagged

Q6

Q7


Q8
0:00:07/03:00:00

A 23-year-old man presents to the clinic for review. He has suffered from two episodes of
balanitis over the past 3 months, and his GP has checked a fasting glucose on two
occasions, measured at 6.2 mmol/l and 6.3 mmol/l. He works on a building site and plays
amateur rugby at weekends. On examination his BP is 148/84; pulse is 80 and regular. His
BMI is 32 and a recent HbA1c checked by his GP is 40 mmol/l (5.8%).

A Impaired fasting glucose

B Impaired glucose tolerance

C Maturity onset diabetes of the young

D Type 2 diabetes

E Type 1 diabetes

Explanation 

A Impaired fasting glucose

The fasting glucose is abnormal under WHO criteria, (6.1-6.9 mmol/l is defined as impaired
fasting glucose (IFG), but below the level for defining Type 2 diabetes, (7.0 mmol/l or
greater). In this situation IFG marks out increased risk of developing T2DM, and increased
risk of ischaemic cardiovascular disease.

B Impaired glucose tolerance

Impaired glucose tolerance is incorrect. This is defined by a glucose between 7.8 and 11
mmol/l 2hrs after an oral glucose tolerance test. Again it is associated with increased risk of
Type 2 diabetes and ischaemic cardiovascular disease.

C Maturity onset diabetes of the young

Maturity onset diabetes of the young, (MODY) is incorrect. MODY patients are usually of
normal weight at the point of diagnosis.
D Type 2 diabetes

Type 2 diabetes is incorrect. The fasting glucose does not fit with a diagnosis of T2DM, nor
does the HbA1c, which is in the “pre-diabetes” range.

E Type 1 diabetes

Type 1 diabetes is incorrect. There are no symptoms of insulinopaenia and the patient is
obese, both counting against a diagnosis of T1DM.
41101

Rate this question:

Next Question

Previous Question Tag Question

Feedback End Review

Difficulty: Average

Peer Responses %

Q. Answered Flagged

Q1

Q2

Q3

Q4

Q5

Q6
Q. Answered Flagged

Q7

Q8

0:00:07/03:00:00

A 72-year-old man who has worked as a gamekeeper on a country estate comes to the clinic
complaining of hearing loss. He complains in particular that he cannot differentiate speech
against a background of other ambient noise. Apart from a past history of hypertension, he is
otherwise completely well. Examination in the clinic reveals a BP of 125/80 mmHg, pulse is
75/min and regular. There is no focal neurology, pure tone audiometry reveals bilateral high
frequency hearing loss.

A Loss of cochlear hair cells

B Otosclerosis

C Presbycusis

D Vestibular schwannoma

E Wax accumulation

Explanation 

A Loss of cochlear hair cells

This person’s profession is the clue to the underlying cause of hearing loss, since in working
as a game keeper he is likely to have been exposed to shot gun sounds on multiple
occasions. Multiple episodes of barotrauma lead to damage to cochlear hair cells, and this is
the most likely pathophysiology here. Hearing loss can be prevented with adequate ear
protection, either it was not available during this patient’s early working life, or he has been
poorly compliant with recommendations to use ear protection.

B Otosclerosis

Otosclerosis is incorrect. Otosclerosis also leads to hearing loss, although it is thought to


occur in relation to previous measles infection, not to noise related damage as is seen here.

C Presbycusis
Presbycusis is incorrect. Presbycusis is age related epithelial atrophy, and would be the
primary differential if we were unaware of this patient’s occupation.

D Vestibular schwannoma

Vestibular schwannoma is incorrect. Vestibular schwannoma leads to unilateral hearing loss


and tinnitus.

E Wax accumulation

Wax accumulation is incorrect. Wax accumulation would be identified at examination, and


leads to conduction hearing loss.
40967

Rate this question:

Next Question

Previous Question Tag Question

Feedback End Review

Difficulty: Average

Peer Responses %

Q. Answered Flagged

Q1

Q2

Q3

Q4
Q. Answered Flagged

Q5

Q6

Q7

Q8 
0:00:07/03:00:00

A 70-year-old woman is admitted to the Oncology Unit for further investigation. She
presented to the Emergency Department after a fall in which she sustained a fracture of
her left femur. On further investigation it was thought to be pathological, and a bone scan
revealed hot spots in the lumbar spine, the opposite femur and the right humerus. She is
a non-smoker with a history of asthma and hypertension.

A Bladder

B Breast

C Colon

D Lung

E Kidney

Explanation 

B Breast

In females breast and lung primaries account for 80% of patients presenting with bony
metastases. In this situation careful examination of the breasts is essential, with consideration
of USS / mammography to rule out any underlying lesion if physical exam is equivocal. The
lack of a history of smoking or significant chest symptoms counts against lung cancer, the
other most prominent cause of bony metastases.

A Bladder

Bladder is incorrect. Bladder cancer is less likely to metastasise to bone, locally invasive
disease is more common.

C Colon

Colon is incorrect. Metastases from colon cancer account for less than 20% of secondary
bone cancers in women.
D Lung

Lung is incorrect. Although bony metastasis from lung cancer, in combination with breast
cancer metastases, make up 80% of secondary bone cancer cases in women, the lack of
chest symptoms and non-smoking history count against the diagnosis here.

E Kidney

Kidney is incorrect. Kidney cancer is part of the 20% of secondary bone cancers that are not
due to breast or lung primaries.
40966

Rate this question:

Next Question

Previous Question Tag Question

Feedback End Review

Difficulty: Average

Peer Responses %

Q. Answered Flagged

Q1

Q2

Q3

Q4

Q5
Q. Answered Flagged

Q6

Q7


Q8
0:00:07/03:00:00

A 72-year-old woman is admitted with ureteric colic. There is a past history of epilepsy but
she has been fit-free for nearly 15 years. This patient has suffered multiple renal stones
and has chronic renal impairment with a serum creatinine of 210 µmol/l. You prescribe
pethidine for pain relief and she initially settles. However, you are asked to see her
urgently on call the following morning as she has suffered a generalised seizure.

A Accumulation of morphine 6-glucuronide after pethidine administration

B Accumulation of norpethidine leading to toxicity

C Accumulation of pethidine leading to toxicity

D An epileptic fit in keeping with her history of previous epilepsy

E Pseudoseizure

Explanation 

B Accumulation of norpethidine leading to toxicity

Pethidine is metabolised to norpethidine, but in patients with renal impairment norpethidine


accumulates rather than being excreted through the kidneys. Norpethidine is toxic and is
associated with a risk of seizures.

A Accumulation of morphine 6-glucuronide after pethidine administration

Accumulation of morphine 6-glucuronide after pethidine administration is incorrect.


Morphine 6-glucuronide accumulates in renal failure when patients are treated with
morphine, increasing the likelihood of opiod toxicity, but this is irrelevant to administration
of pethidine.

C Accumulation of pethidine leading to toxicity

Accumulation of pethidine leading to toxicity is incorrect. It is the metabolite of pethidine


rather than pethidine itself that causes seizures.
D An epileptic fit in keeping with her history of previous epilepsy

An epileptic fit in keeping with her history of previous epilepsy is incorrect. Seizures happen
in patients with epilepsy, but this is a less satisfactory explanation in this situation, as
norpethedine will have provoked seizure.

E Pseudoseizure

Pseudoseizure is incorrect. There is no association between pethidine and pseudoseizures.


46833

Rate this question:

Next Question

Previous Question Tag Question

Feedback End Review

Difficulty: Average

Peer Responses %

Q. Answered Flagged

Q1

Q2

Q3

Q4

Q5

Q6
Q. Answered Flagged

Q7

Q8

0:00:07/03:00:00

A 24-year-old man presents to the Neurology Clinic for review. He reports loss of position
and vibration sense in his arms and legs, clumsiness such that he often falls, and accusations
from friends and work colleagues that he often seems drunk. He tells you that his father
suffered from similar symptoms from his early 40s, and died a few years later with
cardiovascular disease. He is a non-smoker who drinks only occasional alcohol. Examination
reveals distal motor weakness involving the feet and ankles, loss of deep tendon reflexes,
and loss of vibration sense and proprioception bilaterally. There is obvious cerebellar ataxia.
Nerve conduction studies suggest an axonal neuropathy.

A Charcot Marie Tooth disease

B Friedreich’s ataxia

C Huntington’s chorea

D Refsum’s disease

E Spinocerebellar ataxia Type 1

Explanation 

B Friedreich’s ataxia

The progressive neurological symptoms, with loss of sensation followed by progressively


worsening cerebellar ataxia, and distal limb weakness fit well with a diagnosis of FA.
Although Friedreich's is a trinucleotide repeat disorder, it doesn't usually show genetic
anticipation due to autosomal recessive inheritance. Genetic testing for triplet repeat
expansions in the first intron of the frataxin gene that cause Friedreich’s ataxia should be
performed in all patients with suspected FA.

A Charcot Marie Tooth disease

Charcot Marie Tooth disease (CMT) is incorrect. Although symptoms of CMT may be similar,
it is a demyelinating neuropathy.
C Huntington’s chorea

Huntington’s chorea is incorrect. Huntington’s is characterised by early mood disturbance


and potentially depression, and choreiform movements, both of which are absent here.

D Refsum’s disease

Refsum’s disease is incorrect. Refsum’s is associated with retinitis pigmentosa and ichthyosis,
neither of which is seen here.

E Spinocerebellar ataxia Type 1

Spinocerebellar ataxia Type 1 (SCA1) is incorrect. Although symptoms of SCA1 are similar
to FA, FA is much more common, and SCA1 tends to present in late childhood / young
adulthood.
41154

Rate this question:

Next Question

Previous Question Tag Question

Feedback End Review

Difficulty: Average

Peer Responses %

Q. Answered Flagged

Q1

Q2
Q. Answered Flagged

Q3

Q4

Q5

Q6

Q7

Q8 
0:00:07/03:00:00

A 24-year-old theatre nurse presents for review. She has red, scaly plaques that are worst
on her hands, but are also present on the flexor surfaces of her arms and legs. She
complains of a lot of itching. Her past history of note includes coeliac disease.

A Atopic eczema

B Dermatitis herpetiformis

C Lichen planus

D Psoriasis

E Subacute cutaneous lupus erythematosus

Explanation 

A Atopic eczema

Atopic eczema is very common, with a prevalence of up to 10–20% in


children. Some 85% of patients have disease onset before the age of 5
years.
In children who develop eczema there is a co-association with asthma or
allergic rhinitis in up to 50%.
Eczema tends to begin on the cheeks in infants and may become generalised.
In older children and some adults, lesions characteristically affect flexural surfaces, but
may not be restricted to these locations.
Atopic eczema is associated with impaired skin barrier function; it therefore often co-
exists with irritant contact dermatitis, which may be triggered and exacerbated by
frequent handwashing or wet work.

Treatment

Regular application of an emollient is essential; some are also suitable as wash


products to allow avoidance of irritation from soap.
A topical corticosteroid is often necessary: this should be of a potency appropriate to
the site of disease and age of the patient; once-daily application is usually sufficient
and treatment is often recommended in bursts of perhaps 5–7 days.
A short course of oral prednisolone may be considered in severe cases (e.g. 20 mg per
day with weekly decrements over 4 weeks).
Ciclosporin, azathioprine, mycophenolate mofetil and methotrexate can be used
as steroid-sparing agents.

B Dermatitis herpetiformis

Dermatitis herpetiformis is incorrect. Dermatitis herpetiformis results in intensely itchy


vesicles especially on extensor surfaces. These tend to be quickly excoriated. Scaly plaques
are not a feature.

C Lichen planus

Lichen planus is incorrect. Lichen planus classically presents with pruritic, violaceous papules.
These are frequently seen on the flexor aspect of the wrists and around the ankles. Scaly
plaques are not a feature.

D Psoriasis

Psoriasis is incorrect. Red, scaly plaques are certainly typical of psoriasis; however, they
tend not to be severely pruritic, which is much more suggestive of eczema. In most cases,
psoriasis preferentially affects extensor surfaces rather than flexural areas.

E Subacute cutaneous lupus erythematosus

Subacute cutaneous lupus erythematosus is incorrect. Subacute cutaneous lupus


erythematosus usually presents with annular, scaly plaques at sun-exposed sites. It tends
not to be very pruritic. The scenario does not suggest a photodistributed eruption.
46408

Rate this question:

Next Question

Previous Question Tag Question

Feedback End Review

Difficulty: Difficult
Peer Responses %

Q. Answered Flagged

Q1

Q2

Q3

Q4

Q5

Q6

Q7

Q8

Q9

0:00:07/03:00:00

An elderly woman presented with malaise for 3 months. There is a history of rheumatoid
arthritis. She had vitiligo and a 2 cm splenomegaly. Her blood picture was: haemoglobin low,
total white count reduced, platelet count reduced, mild macrocytosis, lactate dehydrogenase
(LDH) 362 U/l (240–600).

A Autoimmune haemolytic anaemia

B B12 deficiency

C Hypersplenism

D Myelodysplasia

E Pernicious anaemia

Explanation 

C Hypersplenism

This woman has rheumatoid arthritis, and the combination of pancytopenia with
splenomegaly points towards Felty syndrome as the most likely cause.

A Autoimmune haemolytic anaemia

Autoimmune haemolytic anaemia is incorrect. Auto-immune haemolytic anaemia could


cause splenomegaly, but the LDH level is not raised, and this would not result in a low
platelet count.

B B12 deficiency

B12 deficiency is incorrect. B12 deficiency can be associated in an overlap syndrome in


combination with other auto-immune disorders, but would not explain the splenomegaly.

D Myelodysplasia
Myelodysplasia is incorrect. Myelodysplasia has no particular connection with rheumatoid
arthritis, but can cause a pancytopenia. It is not, however, usually associated with
splenomegaly.

E Pernicious anaemia

Pernicious anaemia is incorrect. Pernicious anaemia can be associated in an overlap


syndrome in combination with other auto-immune disorders, but would not explain the
splenomegaly.
45487

Rate this question:

Next Question

Previous Question Tag Question

Feedback End Review

Difficulty: Difficult

Peer Responses %

Q. Answered Flagged

Q1

Q2

Q3

Q4

Q5

Q6
Q. Answered Flagged

Q7

Q8
0:00:07/03:00:00

A 67-year-old man with a history of ischaemic heart disease is admitted to the Emergency
Department having suffered a cardiac arrest. According to relatives he has suffered from
diarrhoea for the past 4 days, has not drunk properly, but has continued to take his diuretics
and ACE inhibitor medication. The crew report that he was in pulseless VT when they
arrived, and he attained sinus rhythm after a single shock. On examination in the
department he is unconscious, his BP is 100/70 mmHg, pulse is 85/min. A 12 lead ECG reveals
tall peaked T waves and lost P waves.

A IV Amiodarone

B IV Calcium gluconate

C IV Glucose

D IV Magnesium

E IV Potassium

Explanation 

B IV Calcium gluconate

This patient has ECG features of hyperkalaemia, the most likely cause being pre-renal failure
because of continued diuretic and ACE inhibitor use against a background of diarrhoea. A
blood gas sample to check serum potassium is essential, with IV calcium gluconate being the
first step with respect to cardioprotection. This can be followed with insulin and dextrose as
needed, and steps to ensure rehydration.

A IV Amiodarone

IV Amiodarone is incorrect. Although this is an option for recurrent VT, in this situation it will
not impact on the underlying cause, likely hyperkalaemia.

C IV Glucose
IV Glucose is incorrect, although insulin and dextrose may be used to reduce potassium.

D IV Magnesium

IV Magnesium is incorrect. IV magnesium can be considered for recurrent VT where


potassium is in the normal range.

E IV Potassium

IV Potassium is incorrect. The ECG features seen here are consistent with hyperkalaemia, not
hypokalaemia.
41157

Rate this question:

Next Question

Previous Question Tag Question

Feedback End Review

Difficulty: Average

Peer Responses %

Q. Answered Flagged

Q1

Q2

Q3

Q4

Q5
Q. Answered Flagged

Q6

Q7

Q8

0:00:07/03:00:00

A 19-year-old man presents to the Haematology Clinic for review. His clotting was assessed
after a knee injury and haematoma from playing soccer, and he was found to have a mildly
elevated APTT when assessed pre-op for an arthroscopy. There has been no history of
significant bleeding episodes in the past, and he had an uneventful appendicectomy aged 12.
He takes no regular medication. General physical exam is unremarkable.

Investigations:

Hb 12.9 g/dl

WCC 7.8 x109/l

PLT 128 x109/l

Na+ 137 mmol/l

K+ 4.5 mmol/l

Creatinine 85 micromol/l

APTT 48.2 s

PT 12.3 s

Factor VIII lower end of the normal range

A Acquired haemophilia A

B Factor VII deficiency

C Haemophilia B

D Idiopathic thrombocytopaenic purpura

E Von Willebrand’s disease

Explanation 

E Von Willebrand’s disease


Symptoms from VWD may be quite mild, and patients can often undergo surgery with no
serious consequences given modern surgical procedures that minimise blood loss. In this
case the APTT is only slightly above the normal range, and the factor VIII at the lower end
of the normal range is supportive of the diagnosis. The main treatment options for VWD
are desmopressin and factor VIII concentrates that contain VWF.

A Acquired haemophilia A

Acquired haemophilia A is incorrect given the absence of underlying autoimmune pathology,


and that only minor episodes of bleeding have been seen so far.

B Factor VII deficiency

Factor VII deficiency is incorrect. Factor VII deficiency is a rare autosomal recessive disorder,
associated with a prolonged PT rather than a prolonged APTT.

C Haemophilia B

Haemophilia B is incorrect. Although haemophilia B is associated with a prolonged APTT it is


much rarer versus Haemophilia A and VWD.

D Idiopathic thrombocytopaenic purpura

Idiopathic thrombocytopaenic purpura, (ITP) is incorrect. It is associated with bleeding only


when the platelet count falls to much lower levels versus the 128 x109/l seen here.
41163

Rate this question:

Next Question

Previous Question Tag Question

Feedback End Review

Difficulty: Average

Peer Responses %
Q. Answered Flagged

Q1

Q2

Q3

Q4

Q5

Q6

Q7

Q8

Q9

0:00:07/03:00:00

A 34-year-old man who is HIV positive and poorly compliant with his anti-retroviral
medication comes to the Dermatology Clinic for review. He has developed a number of
red/purple papular lesions, particularly on his arms and legs, back, on his face and in his
mouth. Biopsy of one of the lesions reveals spindle cells with moderately increased mitosis.

A Human herpes virus 1 infection

B Human herpes virus 2 infection

C Human herpes virus 5 infection

D Human herpes virus 6 infection

E Human herpes virus 8 infection

Explanation 

E Human herpes virus 8 infection

HHV8 is also known as the Kaposi’s sarcoma associated virus, the most likely underlying
cause of the lesions seen here. It belongs to the gamma-herpes virus sub family, and has
been seen not only in Kaposi’s, but also in HIV associated lymphoma and Castleman’s
disease (giant lymph node hyperplasia). Highly active antiretroviral therapy (HAART) is well
recognised to limit development of Kaposi’s, and is therefore the most important step in
preventing its development.

A Human herpes virus 1 infection

Human-herpes virus 1 is incorrect. HHV1 is a herpes simplex virus that leads to the
development of cold sores.

B Human herpes virus 2 infection

Human herpes virus 2 is incorrect. HHV2 is a herpes simplex virus that leads to the
development of genital herpes.
C Human herpes virus 5 infection

Human herpes virus 5 is incorrect. HHV5 is cytomegalovirus, which is associated with various
presentations in HIV infected patients including abnormal LFTs, pneumonia, retinitis and
encephalitis.

D Human herpes virus 6 infection

Human herpes virus 6 is incorrect. HHV6 is known as a cause of “6th” disease in children.
40986

Rate this question:

Next Question

Previous Question Tag Question

Feedback End Review

Difficulty: Average

Peer Responses %

Q. Answered Flagged

Q1

Q2

Q3

Q4

Q5

Q6
Q. Answered Flagged

Q7

Q8

0:00:07/03:00:00

A 62-year-old man comes to the Neurology Clinic for review. He has noticed increasing
proximal weakness over the past few months, such that he is now barely able to get up
from a low chair, and is only able to climb the stairs once per day. He has a history of
COPD and continues to smoke 20 cigarettes per day. He reports a chronic dry cough, which
has increased in frequency over the past 6 months in particular. His BP is 138/82 mmHg;
pulse is 75/min (atrial fibrillation). There is wheeze and bilateral poor air entry on
auscultation of the chest. BMI is 22. You can demonstrate mild proximal muscle weakness
on neurological exam.

Investigations:

Hb 10.9 g/dl

WCC 10.3 x109/l

PLT 181 x109/l

Na+ 134 mmol/l

K+ 4.3 mmol/l

Creatinine 110 micromol/l

ALT 119 U/l

ALP 162 U/l

Bilirubin 13 micromol/l

Albumin 38 g/l

CXR left hilar mass

A Anti-ACh receptor

B Anti-Ma2

C Anti-Ri

D Anti-Yo

E Anti-VGCC
Explanation 

E Anti-VGCC

The symptoms here, with proximal muscle weakness against a background of probable
bronchial carcinoma, raise the possibility of Lambert Eaton myaesthenic syndrome (LEMS).
LEMS is associated with anti-VGCC antibodies. They are found in less than 5% of patients
with myasthenia gravis, a possible differential here.

A Anti-ACh receptor

Anti-Ach receptor is incorrect. These antibodies are found in patients with myasthenia gravis,
and are only occasionally seen in low titre in patients with LEMs.

B Anti-Ma2

Anti-Ma2 is incorrect. Anti-Ma2 antibodies are found in patients with testicular carcinoma.

C Anti-Ri

Anti-Ri is incorrect. These antibodies are associated with opsoclonus myoclonus syndrome.

D Anti-Yo

Anti-Yo is incorrect. Anti-Yo antibodies are seen in patients with paraneoplastic cerebellar
degeneration.
41106

Rate this question:

Next Question

Previous Question Tag Question

Feedback End Review

Difficulty: Average

Peer Responses %
Q. Answered Flagged

Q1

Q2

Q3

Q4

Q5

Q6

Q7

Q8

Q9

0:00:07/03:00:00

A 43-year-old woman presents to the clinic for review with a rash affecting both shins. She
says this has slowly got worse over the course of a few weeks, and wonders if it could be
related to medication started by her GP. There is a history of Type 2 diabetes,
hypertension, and gastro-oesophageal reflux disease. On examnation her BP is 138/88
mmHg, pulse is 76/min and regular. Her BMI is 32. There are raised red/purple nodules on
both shins.

A Metformin

B Omeprazole

C Ramipril

D Sitagliptin

E Simvastatin

Explanation 

B Omeprazole

The skin rash here is consistent with erythema nodosum (EN). Commonly prescribed
medications that have a plausible link to EN include Omeprazole, hepatitis B vaccination and
Isotretinoin. Sulfonamides are also listed as potential causes of erythema nodosum, although
the link is a little more tenuous versus Omeprazole.

A Metformin

Metformin is incorrect. It has been reported to cause generalised erythematous rashes,


pruritis and urticaria, but not specifically erythema nodosum.

C Ramipril

Rampiril is incorrect. Maculopapular rashes are commonly reported with Ramipril use;
angioedema is of course also uncommonly reported.
D Sitagliptin

Sitagliptin is incorrect. Skin rashes very rarely occurred in the phase 3 program. Review of
the prescribing information recognises occurrences of angioedema and cutaneous vasculitis,
but the frequency is too low to quantify accurately.

E Simvastatin

Simvastatin is incorrect. Itching and alopecia are rarely reported in conjunction with
Simvastatin therapy, not erythema nodosum specifically.
41019

Rate this question:

Next Question

Previous Question Tag Question

Feedback End Review

Difficulty: Average

Peer Responses %

Q. Answered Flagged

Q1

Q2

Q3

Q4

Q5
Q. Answered Flagged

Q6

Q7


Q8
0:00:07/03:00:00

A 72-year-old man with a previous right-sided squamous cell lung cancer presents to the
Emergency Department with stridor, a severe headache and upper limb and facial oedema,
which has developed rapidly over the past 2 weeks. He received radical radiotherapy, rather
than surgical resection, for a right-sided tumour some 1 year earlier. On examination he has
marked shortness of breath and stridor, is complaining of a severe headache, and has
obvious facial and upper limb oedema with a ruddy complexion. BP measured in both arms
is elevated at 155/95 mmHg. Auscultation of the chest reveals coarse crackles and wheeze
bilaterally with decreased right-sided breath sounds. An urgent CT thorax reveals a right-
sided tumour recurrence with SVC compression and invasion.

A CT guided tumour biopsy and targeted chemotherapy

B Endovascular stenting

C IV corticosteroids

D Surgical debulking

E Urgent radiotherapy

Explanation 

B Endovascular stenting

This patient has significant symptoms related to SVC obstruction and a limited life
expectancy. The goal of intervention is rapid symptom relief, and stenting is therefore the
logical next step. Radiotherapy first may lead to significant scarring, making future stenting
technically difficult, therefore stenting should always be considered first.

A CT guided tumour biopsy and targeted chemotherapy

CT guided tumour biopsy and targeted chemotherapy is incorrect. This is most likely to
represent a squamous cell carcinoma recurrence; as such biopsy is therefore unnecessary.

C IV corticosteroids
IV corticosteroids is incorrect. Steroids will have a limited effect on edema, and therefore buy
minimal time before the need for further intervention.

D Surgical debulking

Surgical debulking is incorrect. Given the technical difficulty of surgery, likely poor functional
status, and limited life expectancy, this is not appropriate.

E Urgent radiotherapy

Urgent radiotherapy is incorrect. Stenting followed by assessment for radiotherapy is the


appropriate sequence of events.
41109

Rate this question:

Next Question

Previous Question Tag Question

Feedback End Review

Difficulty: Average

Peer Responses %

Q. Answered Flagged

Q1

Q2

Q3

Q4
Q. Answered Flagged

Q5

Q6

Q7

Q8 
0:00:07/03:00:00

A 56-year-old man who has recently been diagnosed with Type 2 diabetes but is poorly
compliant with therapy, comes to the Emergency Department for review. He complains of
bilateral blurred vision, and says this happens on a number of occasions during the course
of the week. He is supposed to take Metformin 1g BD, but only does this on weekends
when he is not working as a taxi driver. On examination his BP is 155/90 mmHg; pulse is
78/min and regular. His BMI is 33. Vision is reduced to 6/18 bilaterally, corrected to normal
with his usual glasses. Fundoscopy shows bilateral changes consistent with hypertensive
retinopathy.

Investigations:

Hb 13.1 g/dl

WCC 9.1 x109/l

PLT 171 x109/l

Na+ 137 mmol/l

K+ 4.9 mmol/l

Creatinine 110 micromol/l

Glucose 18.9 mmol/l

HbA1c 76 mmol/mol (9.1%)

A Central retinal artery occlusion

B Central retinal vein occlusion

C Diabetic retinopathy

D Fluctuations in blood glucose

E Presbyopia

Explanation 

D Fluctuations in blood glucose


The most likely cause of this man’s blurred vision is hyperglycaemia, leading to changes in
the shape of the lens related to increased uptake of water. Hypoglycaemia can also lead to
blurred vision. With improved compliance the episodes of blurred vision should resolve.

A Central retinal artery occlusion

Central retinal artery occlusion is incorrect. Much more severe loss in visual acuity is seen,
and changes in the retina, (pale with a “cherry red” spot) are seen.

B Central retinal vein occlusion

Central retinal vein occlusion is incorrect. This results in significant flame shaped
haemorrhages not seen here.

C Diabetic retinopathy

Diabetic retinopathy is incorrect. This is not seen in patients with very short duration
diabetes.

E Presbyopia

Presbyopia is incorrect. Development of long-sightedness would not result in fluctuating


blurred vision.
41162

Rate this question:

Next Question

Previous Question Tag Question

Feedback End Review

Difficulty: Average

Peer Responses %
Q. Answered Flagged

Q1

Q2

Q3

Q4

Q5

Q6

Q7

Q8

Q9

0:00:07/03:00:00

A 56-year-old South Asian woman attends the clinic with her daughter. She complains of
increasing bone and proximal muscle pain, and claims that she has no energy to even go
out of the house. She has a history of hypertension for which she takes Indapamide, and
mild asthma for which she uses a Ventolin inhaler, but is otherwise well. On examination
her BP is 142/84 mmHg; pulse is 77/min and regular. Respiratory and cardiovascular
systems are unremarkable. Abdomen is soft and non-tender, and her BMI is 30. There is
4/5 proximal myopathy affecting both her shoulders and hips.

Investigations:

Hb 11.9 g/dl

WCC 7.4 x109/l

PLT 183 x109/l

Na+ 137 mmol/l

K+ 4.5 mmol/l

Creatinine 105 micromol/l

Ca2+ 2.05 mmol/l

Albumin 41 g/l

ALP 185 U/l

ALT 34 U/l

ESR 21 mm/1st hour

CK 55 U/l

A Calcium and vitamin D

B Prednisolone

C Resistance training

D Thyroxine
E Weight loss

Explanation 

A Calcium and vitamin D

The calcium below the limit of normal, coupled with raised alkaline phosphatase is consistent
with a diagnosis of osteomalacia, which is well recognised to be associated with proximal
myopathy. In this circumstance calcium and vitamin D supplementation should significantly
improve both bone pain and proximal muscle weakness.

B Prednisolone

Prednisolone is incorrect. Prednisolone may be of value in myositis, but there is no reason to


suspect that as the underlying diagnosis here.

C Resistance training

Resistance training is incorrect. Whilst training may help improve muscle strength, without
correcting calcium and vitamin D, it is unlikely to impact significantly here.

D Thyroxine

Thyroxine is incorrect. Although the patient is overweight, there is no evidence that weight
gain here is related to hypothyroidism.

E Weight loss

Weight loss is incorrect. Although weight loss may improve ability to mobilise, it is more
important to correct calcium and vitamin D initially.
41083

Rate this question:

Next Question

Previous Question Tag Question

Feedback End Review

Difficulty: Average
Peer Responses %

Q. Answered Flagged

Q1

Q2

Q3

Q4

Q5

Q6

Q7

Q8

Q9

0:00:07/03:00:00

A 54-year-old man comes to the Emergency Department with a hot, swollen left knee,
which is significantly limiting his range of movement. He is unable to continue his job in a
warehouse, and has seen little improvement despite taking regular paracetamol. On
examination his BP is 125/80 mmHg; pulse is 68/min and regular. He is apyrexial. Knee
flexion is limited to 30 degrees. There is erythema and an effusion. Aspiration demonstrates
positive birefringent rhomboid crystals.

A Calcium oxalate

B Calcium pyrophosphate

C Homogentisic acid

D Magnesium ammonium phosphate

E Uric acid

Explanation 

B Calcium pyrophosphate

The presentation, with a single joint monoarthritis is typical of either gout or


pseudogout, and positively birefringent rhomboid crystals are typical of calcium
pyrophosphate. In this situation physiotherapy coupled with use of NSAIDs is the
mainstay of intervention.

A Calcium oxalate

Calcium oxalate is incorrect. Calcium oxalate crystals are a common constituent of renal
stones, and oxalate forms envelope shaped crystals.

C Homogentisic acid

Homogentisic acid is incorrect. Homogentisic acid is accumulated in joints as a result of


alkaptonuria, caused by a deficiency in homogentisic acid oxidase.
D Magnesium ammonium phosphate

Magnesium ammonium phosphate (MAP) is incorrect. MAP is also known as struvite and is
related to proteus urinary tract infection.

E Uric acid

Uric acid is incorrect. Uric acid crystals are negatively birefringent and needle-
shaped.
40988

Rate this question:

Next Question

Previous Question Tag Question

Feedback End Review

Difficulty: Average

Peer Responses %

Q. Answered Flagged

Q1

Q2

Q3

Q4

Q5

Q6
Q. Answered Flagged

Q7

Q8
0:00:07/03:00:00

A 35-year-old West African woman is admitted to the Emergency Department, according to


her relatives she has become progressively more drowsy and confused over the past few
days and became unresponsive that morning, leading them to call the ambulance. You
understand from her husband that she was also complaining of headaches and loss of power
in her right arm for a few hours before passing out. Further questioning of relatives reveals
that she has suffered from a chronic cough and gradual weight loss over the past few
months. On examination her GCS is 10. She appears to have asymmetry of movement, with
no movement at all of the right arm and limited movement only of the right leg.
Fundoscopy reveals bilateral papilloedema, retinal haemorrhages and cotton wool spots.

Investigations;

Hb 9.4 g/dl

WCC 3.8 x109/l

PLT 122 x109/l

CRP 112 mg/l

Na+ 133 mmol/l

K+ 4.3 mmol/l

Creatinine 105 micromol/l

ALT 105 U/l

ALP 182 U/l

Bilirubin 13 micromol/l

Glucose 5.4 mmol/l

CT head No mass lesion identified

You suspect she may have HIV, and feel that testing is important with respect to guiding her
clinical care.

A Ask the opinion of the GUM clinic with respect to testing

B Her relatives can be asked to give consent for HIV testing


C She should not be tested for HIV under any circumstances

D Test her for HIV as it is in her best interests

E Treat as if she has HIV

Explanation 

D Test her for HIV as it is in her best interests

There is evidence of chronic immuno-suppressive illness, an acute neurological episode


include possible focal neurological deficit, and a systemic inflammatory response, (raised
CRP). All of these, given her origin in West Africa, raise the possibility of HIV infection.
Knowing the HIV test result will both guide possible anti-viral or anti-bacterial therapies,
and instruct as to the point to introduce highly active antiretroviral therapy (HAART).
Significant debate has occurred over the past few years about testing unconscious patients
where there is a needle stick injury, but no such debate exists where testing is in the
patient’s interests.

A Ask the opinion of the GUM clinic with respect to testing

Ask the opinion of the GUM clinic is incorrect. This risks a potential significant delay in
initiating appropriate therapy.

B Her relatives can be asked to give consent for HIV testing

Her relatives can be asked to give consent is incorrect. This risks a potential significant
breach of confidentiality.

C She should not be tested for HIV under any circumstances

She should not be tested for HIV is incorrect. It is clearly in her best interests to treat.

E Treat as if she has HIV

Treat as if she has HIV is incorrect. In the event she is HIV negative this runs the risk of
exposing her to excess risks related to antiviral drug treatment or interventions targeted at
presumed opportunistic infections.
40975

Rate this question:


Next Question

Previous Question Tag Question

Feedback End Review

Difficulty: Average

Peer Responses %

Q. Answered Flagged

Q1

Q2

Q3

Q4

Q5

Q6

Q7

Q8

Q9

0:00:07/03:00:00

You are asked to review a 71-year-old woman who is recovering on the Orthopaedic Ward
after a left knee replacement. She has suffered pyrexia and supra-pubic pain, coupled with
dysuria and frequency over the past 48 hours. Her catheter was removed three days ago. An
MSU taken after the catheter was removed has grown MRSA. Examination reveals a BP of
132/82 mmHg; pulse is 80/min and regular. There is minor supra-pubic tenderness. Her
temperature is 37.8°C.

A Clindamycin

B Doxycycline

C Flucloxacillin

D Linezolid

E Vancomycin

Explanation 

B Doxycycline

Tetracyclines are the first line intervention for MRSA urinary tract infection. Trimethoprim can
be used as an oral alternative, and in patients who have a severe UTI, IV Vancomycin is an
option.

A Clindamycin

Clindamycin is incorrect. Clindamycin is usually employed in the treatment of bone and soft
tissue infections.

C Flucloxacillin

Flucloxacillin is incorrect. MRSA is resistant to Flucloxacillin therapy.

D Linezolid
Linezolid is incorrect. Use of Linezolid monotherapy is discouraged because of the risk of
resistance developing.

E Vancomycin

Vancomycin is incorrect. Vancomycin is reserved for more severe


infection.
40980

Rate this question:

Next Question

Previous Question Tag Question

Feedback End Review

Difficulty: Average

Peer Responses %

Q. Answered Flagged

Q1

Q2

Q3

Q4

Q5

Q6

Q7
Q. Answered Flagged

Q8
0:00:07/03:00:00

A 78-year-old woman who has chronic lymphocytic leukaemia, (CLL) which was diagnosed 3
years earlier, comes to the Emergency Department because of a cough productive of
purulent / blood stained sputum. She has been admitted with pneumonia 3 times over the
course of the last 6 months. On examination she is pyrexial 38.5°C, BP is 122/80 mmHg;
pulse is 90/min and regular. There are signs of right basal consolidation. Abdominal
examination reveals splenomegaly consistent with her underlying diagnosis.

Investigations:

Hb 8.9 g/dl

WCC 61.2 x 109/l

Lymphocytes 56.1 x 109/l

PLT 112 x 109/l

Na+ 137 mmol/l

K+ 5.2 mmol/l

Creatinine 118 micromol/l

CRP 175 mg/l

Albumin 34 g/l

ALT 52 U/l

ALP 132 U/l

Bilirubin 14 micromol/l

CXR Right basal consolidation

A Decreased complement

B Hypogammaglobulinaemia

C Poor neutrophil function

D Poor nutritional status


E T cell lymphopaenia

Explanation 

B Hypogammaglobulinaemia

Although only 10% of patients with CLL have significant hypogammaglobulinaemia, in


those with established disease, this is significant in approximately two thirds. The
hypogammaglobulinaemia is reflected in recurrent bacterial infections, especially affecting
the respiratory tract. Patients can be encouraged to present early for medical intervention
at the onset of respiratory symptoms, and supplied with rescue antibiotics to begin at
home.

A Decreased complement

Decreased complement is incorrect. There is decreased C3b binding to bacteria in patients


with CLL, although the major contributor to decreased bacterial immunity in established
disease is thought to be hypogammaglobulinaemia.

C Poor neutrophil function

Poor neutrophil function is incorrect. A primary neutrophil function disorder is not seen in
patients with CLL, poor bacterial killing is more likely to be related to
hypogammaglobulinaemia.

D Poor nutritional status

Poor nutritional status is incorrect. Albumin is 34, which indicates significant nutritional
impairment is unlikely.

E T cell lymphopaenia

T cell lymphopaenia is incorrect. This is more likely to result in recurrent viral


infection.
41004

Rate this question:

Next Question

Previous Question Tag Question

Feedback End Review


Difficulty: Average

Peer Responses %

Q. Answered Flagged

Q1

Q2

Q3

Q4

Q5

Q6

Q7

Q8

Q9

0:00:07/03:00:00

An 18-year-old man is referred to the Renal Clinic for review. He has been referred by his
GP for review with a history of microscopic haematuria. He has also recently been
diagnosed with sensorineural deafness. Apparently his brother who is 4 years older has the
same symptoms and has now developed problems with his kidney function. On examination
his BP is 142/85 mmHg; pulse is 78/min and regular. His BMI is 22. Abdominal palpation is
normal.

Investigations:

Hb 13.1 g/dl

WCC 7.2 x 109/l

PLT 189 x 109/l

Na+ 137 mmol/l

K+ 4.9 mmol/l

Creatinine 110 micromol/l

Urine blood 1+, protein –

A Alport’s syndrome

B Autosomal dominant polycystic kidney disease

C Chronic reflux nephropathy

D Medullary sponge kidney

E Renal cell carcinoma

Explanation 

A Alport’s syndrome

The picture of sensorineural deafness, coupled with microscopic haematuria, and later
progression to renal impairment is typical of Alport’s syndrome. It is commonly inherited as
an X-linked disorder, with females affected, but to a much lesser extent than males.
Control
of blood pressure is key in delaying progression of renal failure. In patients who undergo
transplant, up to 5% may develop anti-GBM nephritis.

B Autosomal dominant polycystic kidney disease

Autosomal dominant polycystic kidney disease is incorrect. The lack of palpable kidneys
counts against the diagnosis.

C Chronic reflux nephropathy

Chronic reflux nephropathy is incorrect. This is commoner in women, and usually associated
with a history of urinary tract infection.

D Medullary sponge kidney

Medullary sponge kidney is incorrect. This is cystic dilatation of the collecting tubules in one
or both kidneys, and is associated with increased risk for renal stones.

E Renal cell carcinoma

Renal cell carcinoma is incorrect. This patient is young to present with renal cell carcinoma,
and the family history counts more towards an inherited condition such as Alport’s.
41015

Rate this question:

Next Question

Previous Question Tag Question

Feedback End Review

Difficulty: Average

Peer Responses %
Q. Answered Flagged

Q1

Q2

Q3

Q4

Q5

Q6

Q7

Q8

Q9

0:00:07/03:00:00

A 23-year-old man has returned from travelling in South-East Asia, and presents to the
Emergency Department with nausea, lethargy and jaundice. He says he suffered from a
diarrhoeal illness some 2-3 weeks earlier, which was followed by dull right upper quadrant
pain and anorexia, which has continued until now. He has no past medical history of note
and was vaccinated before his trip, including against hepatitis A and typhoid. On examination
his BP is 122/80 mmHg; pulse is 85/min and regular. Temperature is 37.8°C. He has jaundice
with evidence of scratch marks over the chest and abdomen. There is right upper quadrant
tenderness and a palpable liver edge.

Investigations:

Hb 11.5 g/dl

WCC 11.9 x109/l

PLT 145 x109/l

Na+ 137 mmol/l

K+ 4.0 mmol/l

Creatinine 110 micromol/l

ALT 1315 U/l

ALP 260 U/l

Bilirubin 50 micromol/l

A Hepatitis A

B Hepatitis B

C Hepatitis C

D Hepatitis D

E Hepatitis E

Explanation 
E Hepatitis E

Hepatitis E is transmitted via the faeco-oral route and has a similar clinical picture to
hepatitis A, although no vaccine is currently available for it. Within endemic areas it is
spread via the faeco-oral route, through contaminated water, endemic areas include the
South East Asia countries. Almost all immunocompetent individuals clear hepatitis E without
long-term sequellae. In immunosuppressed individuals both ribavirin and interferon alpha
have been used with success.

A Hepatitis A

Hepatitis A is incorrect. Hepatitis A vaccination has a high success rate, making infection
extremely unlikely.

B Hepatitis B

Hepatitis B is incorrect. Hepatitis B is rarely associated with acute hepatitis, and the lack of
injecting drug abuse or unprotected sexual intercourse counts against this as a diagnosis.

C Hepatitis C

Hepatitis C is incorrect. There is no history of exposure to bodily fluids, and hepatitis C is only
rarely associated with symptoms of acute infection.

D Hepatitis D

Hepatitis D is incorrect. Hepatitis D is only seen in patients with pre-existing hepatitis B.


40970

Rate this question:

Next Question

Previous Question Tag Question

Feedback End Review

Difficulty: Average

Peer Responses %
Q. Answered Flagged

Q1

Q2

Q3

Q4

Q5

Q6

Q7

Q8

Q9

0:00:07/03:00:00

A 22-year-old woman is brought to the Emergency Department having suffered a near


collapse after feeling claustrophobic whilst out in the local nightclub. According to her friends
she complained of numbness around her mouth and in her feet, shortly before she fainted.
She has mild asthma for which she takes Salbutamol PRN, and smokes 5 cigarettes per week.
On examination her BP is 105/80 mmHg; pulse is 85/min and regular. Chest is clear.
Respiratory rate is elevated at 28/min. O2 sats are 98% on the pulse oximeter.

Investigations:

pH 7.5

PaO2 11.0 kPa

PaCO2 3.9 kPa

Bicarbonate 22 mmol/l

A Diazepam

B Explanation of the likely diagnosis

C O2 supplementation

D Salbutamol nebuliser

E Urgent lab calcium measurement

Explanation 

B Explanation of the likely diagnosis

This patient’s symptoms are consistent with a diagnosis of acute hyperventilation syndrome.
Often just explanation of the cause of her symptoms, coupled with a discussion on relaxation
techniques is enough to abort the episode. In patients with severe symptoms re-breathing
measures may also be helpful.

A Diazepam
Diazepam is incorrect. Usually with appropriate calm explanation of the underlying diagnosis,
sedatives are not required.

C O2 supplementation

O2 supplementation is incorrect. This patient is not O2 deficient, and thus supplementation is


not needed.

D Salbutamol nebuliser

Salbutamol inhaler is incorrect. Whilst there is a history of mild asthma, her O2 sats are
normal, and chest is clear on auscultation.

E Urgent lab calcium measurement

Urgent lab calcium measurement is incorrect. Although hyperventilation results in a reduction


in ionised calcium, and hence drives symptoms of hypocalcaemia, there is no indication for
calcium measurement or supplementation.
41077

Rate this question:

Next Question

Previous Question Tag Question

Feedback End Review

Difficulty: Average

Peer Responses %

Q. Answered Flagged
Q. Answered Flagged

Q1

Q2

Q3

Q4

Q5

Q6

Q7

Q8 
0:00:07/03:00:00

A 72-year-old woman with a long history of COPD comes to the clinic for review. She also
has a history of hypertension, and takes a number of medications including high dose
Seretide, Bisoprolol, Atorvastatin and Lisinopril. Other history of note includes recurrent
shoulder dislocation after falls. Her main complaint is of easy bruising affecting her forearms
and both shins. Examination confirms a number of ecchymoses affecting all four limbs.

Investigations:

Hb 12.5 g/dl

WCC 7.1 x109/l

PLT 152 x109/l

PT 12.5 s

APTT 28.1 s

Na+ 137 mmol/l

K+ 4.5 mmol/l

Creatinine 95 micromol/l

Glucose 5.7 mmol/l

A Atorvastatin

B Inhaled Fluticasone

C Platelet dysfunction

D Primary collagen disorder

E Vasculitis

Explanation 

B Inhaled Fluticasone
Inhaled corticosteroids are well recognised to lead to skin fragility and increased propensity
for bruising. Studies suggest that high dose Budesonide leads to decreased collagen
synthesis within the skin, and this may drive the tendency for bruising, in conjunction with
vascular changes.

A Atorvastatin

Atorvastatin is incorrect. Atorvastatin is uncommonly associated with pruritis and an


erythematous rash, and may be associated with alopecia.

C Platelet dysfunction

Platelet dysfunction is incorrect. Platelet count is normal, APTT and PT are both normal,
therefore a bleeding disorder is unlikely. In this age group however, use of aspirin is of
course an important cause of platelet dysfunction.

D Primary collagen disorder

Primary collagen disorder is incorrect. It is unlikely that a primary collagen disorder would
present for the first time at the age of 72 and, although she has recurrent shoulder
dislocation, these are after traumas.

E Vasculitis

Vasculitis is incorrect. Bruising alone is not typical of a vasculitic rash, other symptoms such
as joint disease or Raynaud’s, are also absent.
41026

Rate this question:

Next Question

Previous Question Tag Question

Feedback End Review

Difficulty: Average

Peer Responses %
Q. Answered Flagged

Q1

Q2

Q3

Q4

Q5

Q6

Q7

Q8

Q9

0:00:07/03:00:00

A 28-year-old woman comes for review in the clinic some 3 months after maxillofacial
surgery following an assault, which resulted in damage to the left hand side of her face.
Examination reveals a small surgical scar on the lateral side of the left orbit. When the
corneal reflex is examined, there is an absent direct response respect to the left eye, and an
intact consensual response in the left eye when the right eye is examined.

A III

B IV

C V

D VI

E VII

Explanation 

C V

The examination suggests that the afferent pathway for the left corneal reflex is damaged,
but the efferent pathway is intact. The nasociliary branch of the ophthalmic branch of the
5th cranial nerve is responsible for sensing the stimulus on the cornea, lid, or conjunctiva
and is therefore most likely to have been damaged in the assault.

A III

III is incorrect. The third nerve does not mediate the corneal reflex.

B IV

IV is incorrect. IVth nerve palsies result in superior oblique muscle palsy.

D VI
VI is incorrect. VIth nerve palsies result in lateral rectus palsy.

E VII

VII is incorrect. VIIth nerve palsies result in an efferent corneal reflex defect.

41107

Rate this question:

Next Question

Previous Question Tag Question

Feedback End Review

Difficulty: Average

Peer Responses %

Q. Answered Flagged

Q1

Q2

Q3

Q4

Q5

Q6

Q7

Q8


0:00:07/03:00:00

A 24-year-old man presents to the clinic for review. He tells you that he opens his bowels up
to 6 times per day, has chronic abdominal pain, and often suffers periods of bloating. He also
has tiredness and lethargy and finds it difficult to cope with his job working in a builder’s
yard. He has lost 5kg in weight whilst trying hard to maintain his weight over the past 3
months. His BP is 110/80 mmHg; pulse is 75/min and regular. Abdominal exam reveals mild
central abdominal tenderness, and his BMI is 21.

Investigations:

Hb 11.5 g/dl

MCV 85 fl

WCC 8.1 x109/l

PLT 144 x109/l

Na+ 137 mmol/l

K+ 4.0 mmol/l

Creatinine 95 micromol/l

Albumin 28 g/l

Ca2+ 2.21 mmol/l

ALT 44 U/l

Anti-TTG antibody negative

A Colonoscopy

B Faecal calprotectin

C Faecal fat testing

D SeHCAT test

E Upper GI endoscopy
Explanation 

B Faecal calprotectin

Elevated faecal calprotectin indicates increased migration of neutrophils to intestinal mucosa,


and is therefore a non-invasive screen for inflammatory bowel disease. A normal faecal
calprotectin makes inflammatory bowel disease unlikely. An elevated calprotectin may drive
further bowel imaging.

A Colonoscopy

Colonoscopy is incorrect. There is little indication from the history of large bowel
inflammation; as such non-invasive tests such as calprotectin should be considered first.

C Faecal fat testing

Faecal fat testing is incorrect. It is a relatively non-specific investigation for small bowel
malabsorption, and is difficult to collect for patients, hence it is now rarely performed.

D SeHCAT test

SeHCAT test is incorrect. This is used as a screening test for bile acid diarrhoea.

E Upper GI endoscopy

Upper GI endoscopy is incorrect. Both endoscopy and colonoscopy would be considered


once non-invasive investigations had been concluded.
40998

Rate this question:

Next Question

Previous Question Tag Question

Feedback End Review

Difficulty: Average

Peer Responses %
Q. Answered Flagged

Q1

Q2

Q3

Q4

Q5

Q6

Q7

Q8

Q9

0:00:07/03:00:00

A 32-year-old woman with a history of Hashimoto’s thyroiditis comes to the Emergency


Department by ambulance after collapsing at the local supermarket. She has been treated
with thyroxine replacement 100mcg for the past 4 years, and her TSH has remained around
2.5. Her only other medication is the combined oral contraceptive pill. Examination reveals
a BP of 110/70 mmHg lying, falling to 90/60 mmHg on standing. She is slim and looks
tanned, her BMI is 24.

Investigations:

Hb 12.5 g/dl

WCC 7.9 x109/l

PLT 193 x109/l

Na+ 130 mmol/l

K+ 5.2 mmol/l

Creatinine 95 micromol/l

TSH 2.5 U/l

Glucose 4.7 mmol/l

A Anti-TTG antibodies

B Insulin tolerance test

C Serum osmolality

D Synacthen

E Thyroxine

Explanation 

D Synacthen
Autoimmune hypothyroidism is associated with other autoimmune pathologies including
coeliac disease, Type 1 diabetes and Addison’s disease. The postural hypotension, coupled
with hyponatraemia and potassium at the upper end of the normal range is very suggestive
of Addison’s. A Synacthen test is the logical way to confirm the diagnosis, but should not
delay initiation of corticosteroids. Adrenal insufficiency is excluded by an incremental rise in
cortisol of > 200 nmol/L and a 30 min value > 600 nmol/L.

A Anti-TTG antibodies

Anti-TTG antibodies is incorrect. There are no GI symptoms to suggest an underlying


diagnosis of coeliac disease.

B Insulin tolerance test

Insulin tolerance test is incorrect. ITT is primarily a test for assessing growth hormone
deficiency and is contra-indicated in adrenal insufficiency.

C Serum osmolality

Serum osmolality is incorrect. Given the postural hypotension seen here, SIADH is effectively
ruled out as a diagnosis.

E Thyroxine

Thyroxine is incorrect. TSH has remained stable and this patient is already treated with
thyroxine replacement therapy.
41165

Rate this question:

Next Question

Previous Question Tag Question

Feedback End Review

Difficulty: Average

Peer Responses %
Q. Answered Flagged

Q1

Q2

Q3

Q4

Q5

Q6

Q7

Q8

Q9

0:00:07/03:00:00

A 74-year-old man who has undergone metallic aortic valve replacement some 4 years ago
is admitted to the Cardiology Ward with gradually worsening shortness of breath, decreased
exercise tolerance and lower limb swelling. In addition he has noticed jaundiced sclerae
over the past 2-3 weeks. On examination his BP is 122/82 mmHg; pulse is 85/min and
regular. JVP is elevated, there is pitting oedema affecting both lower limbs and mild ascites.
There is an early diastolic murmur and bilateral inspiratory crackles on auscultation of the
chest. A pulsatile liver is noted on abdominal examination.

A Dilated cardiomyopathy

B Constrictive pericarditis

C Pericardial effusion

D Pulmonary embolus

E Valve failure

Explanation 

E Valve failure

The early diastolic murmur is suggestive of aortic valve back flow because of valve failure, as
is the right heart failure and pulsatile liver. The jaundiced sclerae are suggestive of valve
haemolysis. His right heart failure should be managed with diuresis, and the patient assessed
for valve re-do / replacement.

A Dilated cardiomyopathy

Dilated cardiomyopathy is incorrect. Although this is associated with biventricular failure,


the early diastolic murmur and previous valve replacement point more towards mechanical
valve failure.

B Constrictive pericarditis
Constrictive pericarditis is incorrect. This is associated primarily with right heart failure and is
associated with inflammatory conditions and tuberculosis.

C Pericardial effusion

Pericardial effusion is incorrect. A pericardial effusion is associated with quiet heart sounds,
an low cardiac output, not with the aortic regurgitant murmur heard here.

D Pulmonary embolus

Pulmonary embolus is incorrect. The absence of chest pain, and the presence of other
reasons for heart failure (i.e. the valve replacement), would count against PE as the
underlying problem.
41153

Rate this question:

Next Question

Previous Question Tag Question

Feedback End Review

Difficulty: Average

Peer Responses %

Q. Answered Flagged

Q1

Q2

Q3
Q. Answered Flagged

Q4

Q5

Q6

Q7

0:00:07/03:00:00

You review a 58-year-old man with tuberculosis and commence him on triple antituberculous
therapy. He has a past history of alcoholism. You are aware of the problem of peripheral
neuropathy with isoniazid therapy.

A It occurs because of a defect in butyrylcholinesterase

B It only occurs in alcoholics

C Thiamine prophylaxis is usually used to prevent its occurrence

D Those with an N-acetyltransferase type-1 gene defect are predisposed to


neuropathy

E Those with an N-acetyltransferase type-2 gene defect are predisposed


to neuropathy

Explanation 

E Those with an N-acetyltransferase type-2 gene defect are predisposed


to neuropathy

Thankfully, isoniazid-associated peripheral neuropathy occurs in less than 1% of those who


take the drug, although the incidence is increased at higher drug doses. It is related to a
defect in the gene coding for N-acetyltransferase type 2, resulting in abnormal isoniazid
metabolism and ‘slow acetylator’ status. Isoniazid is also associated with raised
transaminases in 10–20% of those patients for whom it is prescribed, and rarely hepatic
necrosis and jaundice (liver toxicity is more likely to occur in ‘fast acetylator’ status.
Rifampicin and pyrazinamide are two other antituberculous agents that may also be
associated with abnormal liver enzymes.

A It occurs because of a defect in butyrylcholinesterase

It occurs because of a defect in butyrylcholinesterase is incorrect. Butyrylcholinesterase


defects are associated with suxamethonium-induced apnoea. Butyrylcholinesterase is not
relevant to isoniazid metabolism.
B It only occurs in alcoholics

It only occurs in alcoholics is incorrect. It is more common in alcoholics, but not unique to this
patient group.

C Thiamine prophylaxis is usually used to prevent its occurrence

Thiamine prophylaxis is usually used to prevent its occurrence is incorrect. Pyridoxine


(vitamin B6) is given as prophylaxis against peripheral neuropathy occurring, not thiamine
(vitamin B1).

D Those with an N-acetyltransferase type-1 gene defect are predisposed to


neuropathy

Those with an N-acetyltransferase type-1 gene defect are predisposed to neuropathy is


incorrect. NAT-1 mutations may be associated with increased risk of colorectal adenomas.
46877

Rate this question:

Next Question

Previous Question Tag Question

Feedback End Review

Difficulty: Average

Peer Responses %

Q. Answered Flagged

Q1
Q. Answered Flagged

Q2

Q3

Q4

Q5

Q6

Q7

Q8
0:00:07/03:00:00

A 62-year-old woman with a 7-year history of Type 2 diabetes comes to the clinic for
review. Current therapy includes Metformin, Pioglitazone, Aspirin, Lisinopril, Furosemide and
Atorvastatin. She reports gradually increasing weight and increasing peripheral oedema
over the past 4 months. There has been a slow fall off in her exercise tolerance and now
she is only able to walk a few hundred metres before needing to rest. On examination her
BP is 112/85 mmHg; pulse is 80/min and regular. She has pitting oedema to the mid thighs
and evidence of ascites. There are bilateral basal crackles on auscultation of the chest
consistent with heart failure. BMI is 33.

Investigations:

Hb 12.1 g/dl

WCC 9.2 x109/l

PLT 189 x109/l

Na+ 135 mmol/l

K+ 5.1 mmol/l

Creatinine 112 micromol/l

HbA1c 53 mmol/mol (7.0%)

Albumin 31 g/l

ALT 85 U/l

ALP 112 U/l

Bilirubin 14 micromol/l

24hr urinary protein 0.7g

A Carcinomatosis

B Congestive cardiac failure

C Hepatic cirrhosis

D Hypothyroidism
E Nephrotic syndrome

Explanation 

B Congestive cardiac failure

Clues here as to the underlying diagnosis include the relatively short duration of Type 2
diabetes, (less than 10 years), prescription of Pioglitazone, and the fact that 24hr urinary
protein excretion is relatively low at 0.7g/24hrs. This effectively rules out nephrotic
syndrome as the underlying cause. Out of the other options listed, CCF is the most likely.
The Pioglitazone should be discontinued. Options for replacement would include DPPIV
inhibitors, or potentially a GLP-1 agonist if the patient is prepared to accept injectable
therapy.

A Carcinomatosis

Carcinomatosis is incorrect. This is unlikely given that the albumin level is above 30, and
there has been gradual weight gain rather than weight loss.

C Hepatic cirrhosis

Hepatic cirrhosis is incorrect. Although this is a potential alternative diagnosis, apart


from Type 2 diabetes we are not given any other risk factors in the history, and the
presence of Pioglitazone is a better pointer towards heart failure.

D Hypothyroidism

Hypothyroidism is incorrect. Although hypothyroidism can result in fluid retention and


cardiac failure, the pulse rate of 80 and absence of symptoms of hypothyroidism beyond
weight gain, count against the diagnosis.

E Nephrotic syndrome

Nephrotic syndrome is incorrect. The 24hr urinary protein excretion is too low for this to be
nephrotic syndrome.
41170

Rate this question:

Next Question
Previous Question Tag Question

Feedback End Review

Difficulty: Average

Peer Responses %

Q. Answered Flagged

Q1

Q2

Q3

Q4

Q5

Q6

Q7

Q8

Q9

0:00:07/03:00:00

A 28-year-old pianist comes to the Neurology Clinic for assessment of his tremor. He says
that he has problems with a tremor, affecting both hands symmetrically, which is interfering
with his ability to play at concerts. He tells you that he can control the tremor if he really
concentrates, and it is improved if he drinks alcohol. On further questioning it transpires
that his father suffers from a tremor, which was transient at first, but is now there all the
time and is associated with head nodding. Examination confirms a bilateral, symmetrical low
amplitude tremor. Tone and co-ordination are normal.

A Diazepam

B L-dopa

C Propranolol

D Ropinirole

E Topiramate

Explanation 

C Propranolol

The familial benign, low amplitude tremor seen here is entirely consistent with a diagnosis of
essential tremor (ET). Propranolol and Primidone are both effective interventions for
managing the condition. Either intervention is preferred to patients choosing to self medicate
with alcohol. ET tends to be progressive, becoming permanent after a number of years, and
moving from extremities to also involving head nodding.

A Diazepam

Diazepam is incorrect. Benzodiazepines are addictive, and not as effective as Propranolol,


they should therefore be avoided here.

B L-dopa
L-dopa is incorrect. The tremor seen here is not consistent with Parkinson’s disease.

D Ropinirole

Ropinirole is incorrect. Ropinirole is a treatment for Parkinson’s disease.

E Topiramate

Topiramate is incorrect. Topiramate is usually considered for patients with ET who have
symptoms resistant to conventional therapies.
40981

Rate this question:

Next Question

Previous Question Tag Question

Feedback End Review

Difficulty: Average

Peer Responses %

Q. Answered Flagged

Q1

Q2

Q3

Q4

Q5
Q. Answered Flagged

Q6

Q7

Q8
0:00:07/03:00:00

A 72-year-old man who is currently under investigation for probable bronchial carcinoma
comes to the Emergency Department for review because of bilateral lower limb weakness.
He has suffered from gradually increasing back pain over the last 4 weeks, and is now
unable to stand, with altered sensation / pins and needles in both legs. Examination reveals
weakness of hip flexors bilaterally, decreased fine touch and pin prick sensation, and
reduced anal tone. Urgent MRI confirms spinal cord compression from a probable lung
metastasis, with some destruction of the adjacent vertebral body.

A IV Dexamethasone

B Laminectomy

C Oral Prednisolone

D Surgical resection with Methylmethacrylate reconstruction

E Urgent radiotherapy

Explanation 

A IV Dexamethasone

There have been several small studies of corticosteroids in the management of spinal cord
compression as a result of solid tumour metastases. A significantly higher proportion of
patients treated with Dexamethasone in these small studies remain ambulatory at 6 months,
(59 vs 33%), compared to those who are not. With respect to dose, very high doses, (e.g.
100mg/day of Dexamethasone), appear to be associated with significantly greater steroid
related side effects, therefore doses of 16mg / day are usually not used.

B Laminectomy

Laminectomy is incorrect. Studies have shown that laminectomy and radiotherapy confers no
significant advantage versus radiotherapy alone in the management of spinal cord
metastases.
C Oral Prednisolone

Oral Prednisolone is incorrect. Oral Prednisolone is unlikely to impact on local oedema


sufficiently to improve symptoms due to potency.

D Surgical resection with Methylmethacrylate reconstruction

Surgical resection is incorrect. It is unlikely this patient is a candidate for radical surgery.

E Urgent radiotherapy

Urgent radiotherapy is incorrect. Radiotherapy would be the obvious next step, with planning
for therapy as soon as possible after admission.
41108

Rate this question:

Next Question

Previous Question Tag Question

Feedback End Review

Difficulty: Average

Peer Responses %

Q. Answered Flagged

Q1

Q2

Q3
Q. Answered Flagged

Q4

Q5

Q6

Q7

Q8 
0:00:07/03:00:00

A 72-year-old man comes to the clinic for ischaemic heart disease review. He takes a
number of medications for angina and hypertension. You are currently enrolling a new
cardiovascular outcome trial, but patients cannot be taking any medication that leads to
opening of ion channels.

A Amlodipine

B Bisoprolol

C Doxazosin

D Isosorbide mononitrate

E Nicorandil

Explanation 

E Nicorandil

Nicorandil has a dual mode of action leading to relaxation of vascular smooth muscle. Its
potassium channel opening action provides arterial vasodilation, thus reducing afterload,
whilst a secondary nitrate like action promotes venous relaxation and a reduction in preload.

A Amlodipine

Amlodipine is incorrect. Amlodipine inhibits transmembrane calcium influx, so is considered a


channel inhibitor rather than an opener.

B Bisoprolol

Bisoprolol is incorrect. Bisoprolol is a beta-adrenergic receptor antagonist.

C Doxazosin

Doxazosin is incorrect. Doxazosin is an alpha-adrenergic receptor antagonist.


D Isosorbide mononitrate

Isosorbide mononitrate (ISMO) is incorrect. ISMO acts as a nitrate donor, leading to vascular
smooth muscle relaxation.
41055

Rate this question:

Next Question

Previous Question Tag Question

Feedback End Review

Difficulty: Average

Peer Responses %

Q. Answered Flagged

Q1

Q2

Q3

Q4

Q5

Q6

Q7

Q8

Q9

medicines.org.uk/emc/medicine/27352/SPC/Ikorel+10mg+Tablets
(http://www.medicines.org.uk/emc/medicine/27352/SPC/Ikorel+10mg+Tablets)
0:00:07/03:00:00

A 72-year-old man is diagnosed with multiple myeloma. He is started on initial


chemotherapy, including thalidomide, but fails to tolerate the thalidomide treatment. An
autologous stem cell transplant is not planned, because of poor functional status due to
ischaemic heart disease and chronic obstructive pulmonary disease (COPD). On examination
in the clinic his BP is 122/72 mmHg, pulse is 85/min and regular. Auscultation of the chest
reveals coarse wheeze consistent with COPD. There is bilateral ankle swelling. His BMI is 25.

Investigations:

Hb 10.1 g/dl

WCC 8.9 × 109/l

PLT 182 × 109/l

Na+ 138 mmol/l

K+ 4.3 mmol/l

Creatinine 132 µmol/l

A Bortezomib

B Cyclophosphamide

C Doxorubicin

D Lenalidomide

E Melphalan

Explanation 

A Bortezomib

Bortezomib is the most appropriate alternative to thalidomide. This is a proteasome


inhibitor.NICE recommends its use as an alternative to thalidomide when thalidomide
cannot be takendue to side effects or intolerance.
B Cyclophosphamide

Cyclophosphamide is incorrect. Cyclophosphamide is a useful drug that is used alongside


both thalidomide and bortezomib, but it would not be regarded as an alternative to either.

C Doxorubicin

Doxorubicin is incorrect. Doxorubicin is an anthracycline that is not really used in myeloma


therapy.

D Lenalidomide

Lenalidomide is incorrect. Lenalidomide is not the next most appropriate alternative to


thalidomide. This is only recommended for patients with myeloma who have failed two or
more prior therapies, andtherefore is usually used after both thalidomide and bortezomib
have been used.

E Melphalan

Melphalan is incorrect. Melphalan is sometimes used in frail elderly patients with myeloma,
but is inferior to both thalidomide and bortezomib, and is therefore not recommended.
47272

Rate this question:

Next Question

Previous Question Tag Question

Feedback End Review

Difficulty: Difficult

Peer Responses %
Q. Answered Flagged

Q1

Q2

Q3

Q4

Q5

Q6

Q7

Q8

Q9

0:00:07/03:00:00

A 23-year-old man presents to the Emergency Department with severe bleeding related to
disseminated intravascular coagulation. He admits to feeling lethargic and under the weather
for the past 2 months, with night sweats, dull abdominal pain, easy bruising and weight loss.
On examination his BP is 100/70 mmHg; pulse is 93/min and regular. He has marked
splenomegaly on abdominal palpation.

Investigations:

Hb 7.1 g/dl

WCC 10.1 x109/l (blast cells with Auer rods seen on film)

PLT 55 x109/l

PT 42.1 s

APTT 55.2 s

Na+ 137 mmol/l

K+ 5.4 mmol/l

Creatinine 142 micromol/l

ESR 79 mm/1st hour

A Inv (16)

B t (1:22)

C t (8:21)

D t (9:22)

E t (16:16)

Explanation 
C t (8:21)

This translocation is associated with the presence of Auer rods within leukaemic blasts. They
are composed of fused lysosomes and primary neutrophilic granules and contain peroxidase,
lysosomal enzymes, and crystalline inclusions. The translocation was formerly thought to be
limited to patients with acute promyelocytic leukaemia; (AML-M3), however, it is now
recognised to occur in around 7% of patients diagnosed with other forms of AML.

A Inv (16)

Inv (16) is incorrect. This inversion is associated with a degree of monocytic and granulocytic
differentiation in leukaemic cells, and a favourable prognosis with standard therapy.

B t (1:22)

t (1:22) is incorrect. This is rare, accounting for less than 0.5% of patients with AML. It
most commonly occurs in infants.

D t (9:22)

t (9:22) is incorrect. This is the Philadelphia chromosome, most commonly associated with
CML.

E t (16:16)

t (16:16) is incorrect. Similar to the chromosome 16 inversion, the chromosome 16


translocation is associated with a degree of cellular differentiation and a favourable
prognosis.
41030

Rate this question:

Next Question

Previous Question Tag Question

Feedback End Review

Difficulty: Average

Peer Responses %
Q. Answered Flagged

Q1

Q2

Q3

Q4

Q5

Q6

Q7

Q8

Q9

0:00:07/03:00:00

A 40-year-old woman presents with a breast lump, which following biopsy is shown to be
malignant. She tells you that she has a family history of a genetic disorder.

A Ataxia–telangiectasia

B Chédiak–Higashi syndrome

C Cystic fibrosis

D Huntington’s disease

E von Hippel–Lindau disease

Explanation 

A Ataxia–telangiectasia

A number of syndromes cause solid-cancer proneness. Ataxia–telangiectasia is associated


with breast cancer. In ataxia–telangiectasia, the mutated protein (ATM) is a kinase, which
acts as a transducer of DNA-damage signalling, and can also affect binding to BRCA-1 (one
of the two proteins mutated in familial breast cancer). The proteins missing in this
syndrome affect BRCA-1-mediated assembly into multimeric machines responsible for the
surveillance of DNA replication, recombinational repair and the removal of DNA cross-links.

B Chédiak–Higashi syndrome

Chédiak–Higashi syndrome is incorrect. Chédiak–Higashi syndrome is an autosomal recessive


condition associated with the development of lymphoma.

C Cystic fibrosis

Cystic fibrosis is incorrect. Cystic fibrosis is not associated with increased risk of cancer.

D Huntington’s disease
Huntington’s disease is incorrect. Huntington’s is not associated with increased risk of cancer.

E von Hippel–Lindau disease

von Hippel–Lindau disease is incorrect. von-Hippel Lindau, according to whether it is Type


1 or Type 2, may be associated with renal cell carcinoma or phaeochromocytoma.
46477

Rate this question:

Next Question

Previous Question Tag Question

Feedback End Review

Difficulty: Difficult

Peer Responses %

Q. Answered Flagged

Q1

Q2

Q3

Q4

Q5

Q6

Q7
Q. Answered Flagged

Q8
0:00:07/03:00:00

A 32-year-old carpet fitter presents to the Emergency Department with right knee pain and
swelling, such that he is unable to continue his job. He has no past medical history of note
and takes no regular medication. He drinks 8 pints of beer per week and smokes 10
cigarettes per day. Examination reveals a temperature of 37.8°C. There is a 2.5cm swelling
anterior to the patella with overlying erythema and increased temperature. Knee flexion is
significantly limited by pain.

A Diagnostic aspiration of the swelling

B Oral Colchicine

C Oral Flucloxacillin

D Oral Naproxen

E Oral Prednisolone

Explanation 

A Diagnostic aspiration of the swelling

The presentation is consistent with pre-patellar bursitis, also known as “housemaid’s knee”,
an occupational hazard of jobs such as carpet fitting. It is key to establish if the aetiology is
infectious or inflammatory, as such diagnostic aspiration is the next logical step. White
count, protein, lactate and glucose can all help to differentiate between infective and
inflammatory aetiologies. Markedly elevated white count, protein and lactate are strong
pointers to an infective aetiology.

B Oral Colchicine

Oral Colchicine is incorrect. Colchicine is most commonly used for the treatment of gout in
patients who are unable to take NSAIDs or oral steroids for an acute episode.

C Oral Flucloxacillin
Oral Flucloxacillin is incorrect. Blind treatment with antibiotics is inappropriate given an
infective aetiology has not yet been established.

D Oral Naproxen

Oral Naproxen is incorrect. Blind treatment with NSAIDs without establishing whether any
underlying bacterial infection is present is not appropriate.

E Oral Prednisolone

Oral Prednisolone is incorrect. If infection is present, giving systemic corticosteroids may risk
significant local tissue destruction.
41179

Rate this question:

Next Question

Previous Question Tag Question

Feedback End Review

Difficulty: Average

Peer Responses %

Q. Answered Flagged

Q1

Q2

Q3

Q4
Q. Answered Flagged

Q5

Q6

Q7

Q8 
0:00:07/03:00:00

A 64-year-old woman with mild epigastric pain / indigestion, comes to the clinic for review of
her upper GI endoscopy results. She was referred by her GP because of unexplained iron
deficiency anaemia, with an Hb of 8.5g/dl, which had only marginally improved with oral iron
supplementation. On examination in the clinic her BP is 135/80 mmHg; pulse is 85/min and
regular. She looks pale. Abdominal palpation is unremarkable.

Investigations:

Hb 8.5 g/dl

WCC 7.1 x 109/l

PLT 181 x 109/l

Na+ 137 mmol/l

K+ 4.5 mmol/l

Creatinine 90 micromol/l

Upper GI endoscopy one small duodenal ulcer with no active bleeding

A Capsule endoscopy

B Colonoscopy

C Labelled red cell scan

D Omeprazole and review of symptoms in 3 months

E Omeprazole and repeat upper GI endoscopy in 3 months

Explanation 

B Colonoscopy

In this situation, with significant anaemia despite iron replacement, and relatively minor
upper GI pathology, colonoscopy, particularly to rule out a right sided, symptomless colonic
lesion is essential.
A Capsule endoscopy

Capsule endoscopy is incorrect. Capsule endoscopy is usually only considered after normal
upper and lower GI endoscopies when a source of bleeding is not identified.

C Labelled red cell scan

Labelled red cell scan is incorrect. Labelled red cell scanning is usually considered an option
in the evaluation of acute GI bleeding.

D Omeprazole and review of symptoms in 3 months

Omeprazole and review of symptoms in 3 months is incorrect. This does not include essential
imaging of the lower GI tract.

E Omeprazole and repeat upper GI endoscopy in 3 months

Omeprazole and repeat upper GI endoscopy in 3 months is incorrect. Again, this does not
account for imaging of the lower GI tract.
41008

Rate this question:

Next Question

Previous Question Tag Question

Feedback End Review

Difficulty: Average

Peer Responses %
Q. Answered Flagged

Q1

Q2

Q3

Q4

Q5

Q6

Q7

Q8

Q9
0:00:07/03:00:00

A 17-year-old woman presents to the Rheumatology Clinic with facial erythema which
worsens on exposure to the sun, over the past few months she has also developed multiple
pains affecting her fingers, wrists, knees, elbows, and ankles, feels she is off her food, and
has suffered from intermittent night sweats. Her only medication is the oral contraceptive
pill.
Examination reveals a BP of 136/82 mmHg; pulse is 80/min and regular. She has a butterfly
distribution rash affecting her face, and evidence of small joint polyarthritis.

Investigations:

Hb 10.9 g/dl

WCC 9.1 x109/l

PLT 121 x109/l

Na+ 137 mmol/l

K+ 4.8 mmol/l

Creatinine 110 micromol/l

Urine blood +, protein +

CXR clear lung fields, no cardiomegaly

A Anti-CCP antibodies

B Anti-nuclear antibody titre

C C3 levels

D C reactive protein

E Rheumatoid factor titre

Explanation 

C C3 levels
Active SLE (the most likely diagnosis here), results in consumption of complement factors,
including C3 and C4. A fall off in C3 is therefore a useful marker of disease activity in lupus.
Corticosteroids are the initial intervention of choice, followed by Hydroxychloroquine to
control disease activity and act as a steroid-sparing agent.

A Anti-CCP antibodies

Anti-CCP antibodies is incorrect. Anti-CCP antibodies are a marker of rheumatoid arthritis


and their presence is associated with more rapid progression of joint disease.

B Anti-nuclear antibody titre

Anti-nuclear antibody titre is incorrect, although ANAs are associated with a diagnosis of
SLE.

D C reactive protein

C reactive protein is incorrect. It is a non-specific marker of infection or inflammation.

E Rheumatoid factor titre

Rheumatoid factor titre is incorrect. Rheumatoid factor is considered primarily as a marker of


rheumatoid arthritis.
40995

Rate this question:

Next Question

Previous Question Tag Question

Feedback End Review

Difficulty: Average

Peer Responses %
Q. Answered Flagged

Q1

Q2

Q3

Q4

Q5

Q6

Q7

Q8

Q9

0:00:07/03:00:00

A 23-year-old woman is referred to the Endocrine Clinic for review. She has lethargy and
palpitations, and her GP has measured a TSH that is less than 0.05 mU/l. She is 3 months
post partum with her first child, and originally presented to the GP with what she thought
was anxiety and difficulty managing with the first few months of a newborn. On
examination her BP is 112/80 mmHg; pulse is 88/min and regular. She has a fine tremor
and sweaty palms. Neck examination reveals a non-tender thyroid, which is only slightly
enlarged. Eye examination is normal, blood testing reveals anti-TPO antibodies.

A De Quervain’s thyroiditis

B Grave’s disease

C Hashimoto’s thyroiditis

D Iodine deficiency

E Post-partum thyroiditis

Explanation 

E Post-partum thyroiditis

The clues here include the painless nature of the thyrotoxicosis, onset in close proximity to
the recent pregnancy, and the fact that the thyroid is only slightly enlarged. All of these
factors point towards a diagnosis of post-partum thyroiditis. Up to 80% of patients with post-
partum thyroiditis have positive anti-TPO antibodies. The hyperthyroid phase is thought to
last from 2-8 weeks,and is then followed by a period of hypothyroidism that can last up to 6
months.

A De Quervain’s thyroiditis

De Quervain’s is incorrect. This leads to flu-like symptoms and painful thyroiditis.

B Grave’s disease
Grave’s disease is incorrect. Grave’s is associated with eye disease, greater thyroid
enlargement, and pre-tibial myxoedema.

C Hashimoto’s thyroiditis

Hashimoto’s thyroiditis is incorrect. Although post-partum thyroiditis is considered a variant


of Hashimoto’s, the timing makes post-partum thyroiditis much more likely.

D Iodine deficiency

Iodine deficiency is incorrect. Given a well balanced diet, including consumption of sea fish,
iodine deficiency is unlikely.
40964

Rate this question:

Next Question

Previous Question Tag Question

Feedback End Review

Difficulty: Average

Peer Responses %

Q. Answered Flagged

Q1

Q2

Q3

Q4
Q. Answered Flagged

Q5

Q6

Q7

Q8 
0:00:07/03:00:00

A 55-year-old man comes to the clinic for review. He is finding it increasingly difficult to do
his job as a painter, because when he tries to paint with his dominant left hand, the arm
and hand become increasingly painful, weak, pale and mottled, such that he has to rest
every few minutes. The right arm is normal, but he lacks the precision to do his job with the
paintbrush held in that arm. He smokes 10 cigarettes per day, has a history of hypertension
and a previous inferior MI some 10 years earlier. On examination his BP is 145/80 mmHg in
the right arm, 140/80 mmHg in the left arm. When you ask him to lift his arms above his
head, he can only do this for a few seconds before dropping the left arm, and held above
his head, the left radial pulse is absent. You also notice that the left arm quickly becomes
pale and discoloured whilst being held above his head.

Investigations:

Hb 13.1 g/dl

WCC 7.1 x109/l

PLT 201 x109/l

Na+ 137 mmol/l

K+ 4.5 mmol/l

Creatinine 100 micromol/l

Glucose 5.6 mmol/l

Cholesterol 4.5 mmol/l

ESR 10 mm/1st hour

A Brachial artery stenosis

B Cervical radiculopathy

C Left subclavian artery stenosis

D Takayasu’s arteritis

E Thoracic outlet syndrome


Explanation 

E Thoracic outlet syndrome

The presentation is consistent with arterial TOS, with symptoms only present with the arm
held above the head. The cause is likely to be positional occlusion of the left subclavian
artery. Arterial TOS is rare, and is seen in an older population of patients and a higher
percentage of men versus the preponderance of women who suffer from neurogenic TOS.
Assessment with CXR to look for presence of a cervical rib, combined CT scanning with the
arm in the neutral and abducted position are the investigations of choice.

A Brachial artery stenosis

Brachial artery stenosis is incorrect. Positional changes in perfusion of the left arm would not
be associated with brachial artery stenosis.

B Cervical radiculopathy

Cervical radiculopathy is incorrect. This is predominantly associated with neck and


neuropathic pain.

C Left subclavian artery stenosis

Left subclavian artery stenosis is incorrect. The positional nature of this man’s left arm
symptoms implies thoracic outlet syndrome rather than a permanent anatomical subclavian
stenosis.

D Takayasu’s arteritis

Takayasu’s arteritis is incorrect. Takayasu’s is commonest in young men and is associated


with generalised symptoms of vasculitis, not seen here. In addition it is highly unlikely that
symptoms of arterial stenosis would be positional.
41079

Rate this question:

Next Question

Previous Question Tag Question

Feedback End Review

Difficulty: Average
Peer Responses %

Q. Answered Flagged

Q1

Q2

Q3

Q4

Q5

Q6

Q7

Q8

Q9

0:00:07/03:00:00

A 42-year-old man is referred to the Endocrine Clinic for review. The Emergency
Department has referred him after a road traffic accident where he failed to see a
motorbike coming from the side. Visual field testing demonstrated temporal field loss, a
bitemporal hemianopia.
Other past history of note includes erectile dysfunction for which he is currently
undergoing counselling. Examination reveals a BP of 125/80 mmHg; pulse is 73/min and
regular.
Respiratory and cardiovascular exam is normal, Abdomen is soft and non-tender and BMI is
23.

Investigations:

Hb 13.1 g/dl

WCC 7.9 x109/l

PLT 201 x109/l

Na+ 137 mmol/l

K+ 4.5 mmol/l

Creatinine 90 micromol/l

Prolactin 1800 mU/l

A Frontal lobe

B Hypothalamus

C Occipital lobe

D Pituitary

E Temporal lobe

Explanation 

D Pituitary
The bitemporal hemianopia seen here is suspicious of an enlarging pituitary adenoma. With a
modestly raised prolactin, a non-functioning adenoma is a possibility, especially as there are
no other features to suggest acromegaly as an alternative diagnosis. Pituitary MRI coupled
with detailed pituitary function testing are obvious next steps. In the event that a non-
functioning adenoma is proven, it is likely to be non-responsive to medical therapies,
requiring surgical intervention.

A Frontal lobe

Frontal lobe is incorrect. Frontal lobe tumours do not normally result in visual field loss and
are often associated with behavioural disturbance.

B Hypothalamus

Hypothalamus is incorrect. The absence of symptoms of posterior pituitary hormone


dysfunction counts against a hypothalamic tumour.

C Occipital lobe

Occipital lobe is incorrect. Occipital lesions result in cortical blindness.

E Temporal lobe

Temporal lobe is incorrect. Temporal lobe lesions usually result in a right or left superior
homonymous quadrantanopia, depending on the side of the lesion.
40969

Rate this question:

Next Question

Previous Question Tag Question

Feedback End Review

Difficulty: Average

Peer Responses %
Q. Answered Flagged

Q1

Q2

Q3

Q4

Q5

Q6

Q7

Q8

Q9

0:00:07/03:00:00

A 21-year-old woman presents with rapidly deteriorating bone pains, night sweats and
severe lethargy. She also noticed bleeding around her gums and enlarged lymph nodes and
first approached her GP for an HIV test following an episode of unprotected sexual
intercourse whilst on holiday a few months earlier. There is generalised lymphadenopathy on
examination, with splenomegaly, and she is thin with a BMI of 20.

Investigations:

Hb 8.9 g/dl

WCC 20.2 x109/l (numerous atypical lymphocytes on peripheral blood film)

PLT 89 x109/l

Na+ 139 mmol/l

K+ 4.9 mmol/l

Creatinine 112 micromol/l

Albumin 29 g/l

ALT 132 U/l

ALP 210 U/l

Bilirubin 23 micromol/l

CXR Mediastinal mass

Cells are found to be CD20+

Chromosomal analysis reveals 9:22 translocation

A Abnormal LFTs

B Age 21

C Anaemia

D Mediastinal mass
E 9:22 translocation

Explanation 

E 9:22 translocation

In this situation with likely B-cell acute lymphoblastic leukaemia, it is the so called
Philadelphia translocation which is associated with the poorest outcome. This is the most
frequently observed chromosomal re-arrangement in adult ALL, seen in approximately 30%
of cases. Data from 2010 suggests a median event free survival in adults with the
translocation of approximately 8 months. Other poor prognostic indicators include male sex
and white count >50.

A Abnormal LFTs

Abnormal LFTs is incorrect. Abnormal LFTs are frequently observed in patients with ALL and
do not significantly impact on prognosis.

B Age 21

Age 21 is incorrect. Although adults have a worse prognosis compared to children presenting
with ALL, the impact is not as great as the Philadelphia translocation.

C Anaemia

Anaemia is incorrect. Although anaemia is indicative of bone marrow infiltration, it is not


associated with poor prognosis.

D Mediastinal mass

Mediastinal mass is incorrect. Mediastinal lymphadenopathy is common in adults with ALL


and is not a particularly strong prognostic indicator.
41172

Rate this question:

Next Question

Previous Question Tag Question

Feedback End Review


Difficulty: Average

Peer Responses %

Q. Answered Flagged

Q1

Q2

Q3

Q4

Q5

Q6

Q7

Q8

Q9

0:00:07/03:00:00

You are walking with a friend through the local park when you come across an elderly man
who has collapsed. He has no pulse and is making no respiratory effort. Your friend calls for
help.

A 5:1

B 10:2

C 15:1

D 20:2

E 30:2

Explanation 

E 30:2

Chest compressions are much more efficient when breaks are minimised to allow for
breaths to be given. By giving 30 compressions to every 2 breaths this optimises
maintenance of cardiac output and allows adequate oxygenation until help arrives.

A 5:1

5:1 is incorrect. Too frequent breaks for breaths mean that adequate circulation is unlikely to
be maintained.

B 10:2

10:2 is incorrect. Too frequent breaks for breaths mean that adequate circulation is unlikely
to be maintained.

C 15:1
15:1 is incorrect. Too frequent breaks for breaths mean that adequate circulation is unlikely to
be maintained.

D 20:2

20:2 is incorrect. Too frequent breaks for breaths mean that adequate circulation is unlikely
to be maintained.
40978

Rate this question:

Next Question

Previous Question Tag Question

Feedback End Review

Difficulty: Average

Peer Responses %

Q. Answered Flagged

Q1

Q2

Q3

Q4

Q5

Q6

Q7
Q. Answered Flagged

Q8
0:00:07/03:00:00

A 24-year-old man who presented to the Emergency Department with an acute episode of
schizophrenia is treated with Olanzapine. He is reviewed in the Emergency Clinic some 2
months later and wants to discuss potential long-term effects of Olanzapine therapy.

A Abnormal liver enzymes

B Parkinsonism

C Thrombocytopaenia

D Venous thromboembolism

E Weight gain

Explanation 

E Weight gain

Weight gain is very common, occurring in more than 1 in 10 of patients treated with
Olanzapine, with nearly 2/3rds of patients gaining more than 7% of their body weight. Nearly
1/3rd of patients experienced a 15% or greater increase in their body weight. This weight gain
is associated with changes consistent with insulin resistance including abnormal blood lipids,
hypertension and dysglycaemia.

A Abnormal liver enzymes

Abnormal liver enzymes is incorrect. Enzyme changes are common, between 1 in 10 and 1 in
100 of patients treated with Olanzapine.

B Parkinsonism

Parkinsonism is incorrect. Although atypical anti-psychotics are associated with a lower rate
of Parkinson’s symptoms versus traditional anti-psychotics, Parkinsonism is still commonly
seen.
C Thrombocytopaenia

Thrombocytopaenia is incorrect. Thrombocytopaenia is reported in association with


Olanzapine use, although it is a rare occurrence.

D Venous thromboembolism

Venous thromboembolism (VTE) is incorrect. VTE is seen, although it is uncommon.


Increased risk may well be related to weight gain and lower limb venous stasis.
41075

Rate this question:

Next Question

Previous Question Tag Question

Feedback End Review

Difficulty: Average

Peer Responses %

Q. Answered Flagged

Q1

Q2

Q3

Q4

Q5

Q6
Q. Answered Flagged

Q7

Q8

medicines.org.uk/emc/medicine/614#CLINICAL_PRECAUTIONS
(https://www.medicines.org.uk/emc/medicine/614#CLINICAL_PRECAUTIONS)
0:00:07/03:00:00

A 47-year-old woman presented to her general practitioner with chronic fatigue. Liver
function testing was abnormal, with raised aspartate aminotransferase (AST) and alanine
aminotransferase (ALT) levels, accompanied by smaller rises in bilirubin and alkaline
phosphatase levels. Her gamma-globulin was also elevated, and there was a normochromic,
normocytic anaemia. Hepatitis B and C serology were negative, while liver biopsy revealed
chronic hepatitic change.

A Anti-gliadin and anti-endomysial

B Anti-LKM1

C Anti-mitochondrial

D Anti-smooth muscle and anti-nuclear

E Rheumatoid factor

Explanation 

D Anti-smooth muscle and anti-nuclear

This scenario is very suggestive of this woman having type I autoimmune hepatitis; type I
autoimmune hepatitis may present with fatigue, more commonly in women in the peri-
menopausal period, whereas type II autoimmune hepatitis is more likely to present with an
acute episode of jaundice in young women. Type I autoimmune hepatitis is associated with
anti-smooth muscle and anti-nuclear antibody positivity, whereas type II is associated with
anti-LKM1 (‘liver-kidney-muscle 1’) activity; as such, the best choice here is anti-smooth
muscle and anti-nuclear. Treatment of autoimmune hepatitis is typically with prednisolone
during the acute period, followed by a steroid-sparing agent (often azathioprine) long-
term. Corticosteroid and azathioprine therapy induce remission in around 80% of cases.

A Anti-gliadin and anti-endomysial

Anti-gliadin and anti-endomysial is incorrect. Anti-gliadin and anti-endomysial antibodies are


positive in coeliac disease, ruling out this option.
B Anti-LKM1

Anti-LKM1 is incorrect. As described, anti-LKM1 is associated with type II, rather than type I,
autoimmune hepatitis.

C Anti-mitochondrial

Anti-mitochondrial is incorrect. Anti-mitochondrial antibodies are associated with primary


biliary cholangitis, but not with autoimmune hepatitis.

E Rheumatoid factor

Rheumatoid factor is incorrect. Rheumatoid factor is a test for rheumatoid


arthritis.
45828

Rate this question:

Next Question

Previous Question Tag Question

Feedback End Review

Difficulty: Average

Peer Responses %

Q. Answered Flagged

Q1

Q2

Q3
Q. Answered Flagged

Q4

Q5

Q6

Q7

Q8 
0:00:07/03:00:00

A 21-year-old woman with a 16-year history of Type 1 diabetes is admitted to the Emergency
Department having suffered a severe hypoglycaemic episode whilst out shopping. On further
questioning she tells you that she has felt increasingly unwell over the past 2 months with
nausea and lethargy, and has passed out twice at work. Normally her HbA1c is stable at 51
mmol/mol (6.8%), and she uses an insulin pump. She has had to significantly decrease her
pump ratio over the past 2 weeks, but is still experiencing hypos. On examination her BP is
110/80 mmHg, with a postural drop of 20mmHg on standing. She is slim with a BMI of 19.5,
and looks tanned.

Investigations:

Hb 10.9 g/dl

WCC 9.2 x109/l

PLT 171 x109/l

Na+ 131 mmol/l

K+ 4.9 mmol/l

Creatinine 108 micromol/l

Glucose 4.9 mmol/l

HbA1c 41 mmol/mol (5.9%)

A Admit for IV 10% dextrose

B Discharge and tell her to reduce her insulin dose further

C IV hydrocortisone and admit for Synacthen test

D Oral prednisolone and discharge for Outpatient review

E Reassure and tell her no further action is needed

Explanation 
C IV hydrocortisone and admit for Synacthen test

With recent onset hypoglycaemia, despite reducing pump ratios, in a patient with a history
of Type 1 diabetes, an alternative cause to excess exogenous insulin must be suspected. The
sodium is low, and the potassium is in the upper 1/3rd of the normal range, despite
vomiting, taken together with postural hypotension, this makes an Addisonian crisis a
possibility. For this reason IV steroids and admit for Synacthen test is the only possible
option.

A Admit for IV 10% dextrose

Admit for IV 10% dextrose is incorrect. This implies the diagnosis of Addison’s has been
missed.

B Discharge and tell her to reduce her insulin dose further

Discharge and tell her to reduce her insulin dose further is incorrect. Again this risks a
further worsening in her health due to hypoadrenalism.

D Oral prednisolone and discharge for Outpatient review

Oral prednisolone and discharge for Outpatient review is incorrect. It is important to confirm
the diagnosis before committing to long-term steroids.

E Reassure and tell her no further action is needed

Reassure and tell her no further action is needed is incorrect. It is negligent and risks further
hypoglycaemia and potential worsening of the adrenal crisis.
40990

Rate this question:

Next Question

Previous Question Tag Question

Feedback End Review

Difficulty: Average

Peer Responses %
Q. Answered Flagged

Q1

Q2

Q3

Q4

Q5

Q6

Q7

Q8

Q9

0:00:07/03:00:00

A 45-year-old woman who has recently been diagnosed with SLE comes to the clinic for
review. She has noticed increasingly easy bruising over the course of the past few weeks,
and is very concerned that her platelet count may be falling low. Current medication
includes oral prednisolone 40mg/day and she reports continued joint pains and night
sweats. On examination her BP is 142/80 mmHg; pulse is 80/min and regular. There are
bruising and petechial haemorrhages affecting both her arms and legs and she has a
butterfly rash on her face consistent with SLE.

Investigations:

Hb 9.9 g/dl

WCC 4.9 x109/l

PLT 40 x109/l

Na+ 138 mmol/l

K+ 5.1 mmol/l

Creatinine 132 micromol/l

CRP 132 mg/l

ESR 65 mm/1st hour

A Ciclosporin A

B Eltrombopag

C Hydroxychloroquine

D Increased Prednisolone

E Platelet transfusion

Explanation 

C Hydroxychloroquine
Thrombocytopaenia is common in patients with active SLE, with intervention to further
reduce active inflammation the most appropriate next step. Azathioprine, Ciclosporin A and
Hydroxychloroquine are all effective in correcting cytopaenia, although it is
Hydroxychloroquine that is thought to be particularly effective in managing joint disease,
and it does not impact significantly on renal function.

A Ciclosporin A

Ciclosporin A is incorrect because of its potential negative impact on renal function.

B Eltrombopag

Eltrombopag is incorrect, it is only indicated for thrombocytopaenia in ITP and in chronic


hepatitis C.

D Increased Prednisolone

Increased Prednisolone is incorrect. She is already on a relatively high dose, a steroid-sparing


agent such as Hydroxychloroquine is therefore more appropriate.

E Platelet transfusion

Platelet transfusion is incorrect. Platelet transfusion is only usually considered for active
bleeding where the platelet count is less than 20 x109/l.
40996

Rate this question:

Next Question

Previous Question Tag Question

Feedback End Review

Difficulty: Average

Peer Responses %
Q. Answered Flagged

Q1

Q2

Q3

Q4

Q5

Q6

Q7

Q8

Q9

0:00:07/03:00:00

You are investigating a novel immunomodulatory agent for the treatment of Type 1
diabetes, which you think interferes with the way that T cells interact with Class I MHCs.

A CD 4

B CD 5

C CD 8

D CD 19

E CD 20

Explanation 

C CD 8

The extracellular IgV-like domain of CD8-α interacts with the α3 portion of the Class I MHC
molecule. The purpose of MHC class I expression is to identify the cell as virally infected and
therefore drive T cell attack.

A CD 4

CD4 is incorrect. The purpose of the CD4 receptor is to allow communication with antigen
presenting cells, to amplify the immune response to infection, although T helper cells, which
express CD4, may also magnify abnormal, autoimmune responses.

B CD 5

CD5 is incorrect. CD5 primarily serves as a signal modulator on B-lymphocytes.

D CD 19
CD19 is incorrect. The CD19 receptor is found on B-lymphocytes and its function is to reduce
the threshold for antigen dependent B cell stimulation.

E CD 20

CD20 is incorrect. CD20 is a B cell receptor, which is the target of agents such as
Rituximab, an anti-CD20 monoclonal antibody which is B cell depleting.
41160

Rate this question:

Next Question

Previous Question Tag Question

Feedback End Review

Difficulty: Average

Peer Responses %

Q. Answered Flagged

Q1

Q2

Q3

Q4

Q5

Q6

Q7
Q. Answered Flagged

Q8
0:00:07/03:00:00

A 20-year-old man presents to the Emergency Department passing red/brown urine, and
he is concerned that it might be blood. He completed a 1 week course of penicillin V for a S
pyogenes throat infection some 3 days before the haematuria began. He is otherwise well
and takes no regular medication. On examination his BP is 115/85 mmHg; pulse is 75/min
and regular. He is apyrexial. Respiratory, cardiovascular and abdominal examination is
unremarkable.

Investigations:

Hb 12.0 g/dl

WCC 9.8 x 109/l

PLT 168 x 109/l

ESR 60 mm/1st hour

Na+ 137 mmol/l

K+ 4.5 mmol/l

Creatinine 88 micromol/l

Urine red cells +++, protein +

A Henoch-Schönlein purpura

B IgA nephropathy

C Post streptococcal glomerulonephritis

D Scarlet fever

E Urinary tract infection

Explanation 

C Post streptococcal glomerulonephritis


The timing, between 1 and 2 weeks after the onset of symptoms of streptococcal throat
infection, fits with post streptococcal glomerulonephritis, (PSGN). A slightly longer
latency, (3-6 weeks) is seen before symptoms develop after skin infection. Painless
haematuria, (as here), is the typical clinical presentation.

A Henoch-Schönlein purpura

Henoch-Schönlein purpura (HSP) is incorrect. HSP most commonly presents with a


petechial rash affecting the buttocks and the tops of the thighs; this is accompanied by
haematuria, and is commoner in children.

B IgA nephropathy

IgA nephropathy is incorrect. IgA nephropathy is commonly associated with upper


respiratory tract infection, although the interval between infection and symptoms is
commonly less than 5 days.

D Scarlet fever

Scarlet fever is incorrect. Scarlet fever commonly affects children, is associated with sore
throat, fever and an erythematous rash.

E Urinary tract infection

Urinary tract infection is incorrect. The absence of symptoms of UTI would count against
this as an underlying diagnosis.
41013

Rate this question:

Next Question

Previous Question Tag Question

Feedback End Review

Difficulty: Average

Peer Responses %
Q. Answered Flagged

Q1

Q2

Q3

Q4

Q5

Q6

Q7

Q8

Q9

0:00:07/03:00:00

A 43-year-old man with chronic liver disease related to alcoholism comes to the Hepatology
Clinic for review. He maintains that he has minimised his consumption of alcohol and is most
concerned about the cosmetic appearance of his chest. On examination his BP is 105/80
mmHg; pulse is 75/min and regular. He has signs of chronic liver disease and bilateral
gynaecomastia.

A Decreased androstenedione catabolism

B Decreased oestrogen production

C Decreased testosterone production

D Increased HCG production

E Increased testosterone catabolism

Explanation 

A Decreased androstenedione catabolism

Liver dysfunction leads to decreased steroid hormone catabolism. In the case of


androstenedione this means further substrate is then available for peripheral conversion to
oestrogens, leading to gynaecomastia. Overweight, associated with excess calorie intake as a
result of alcoholism, further increases aromatase activity, exacerbating symptoms associated
with feminisation.

B Decreased oestrogen production

Decreased oestrogen production is incorrect. There is potentially an increase in oestrogen


production because of the effects of aromatase on androstenedione.

C Decreased testosterone production

Decreased testosterone production is incorrect. Lack of testosterone is not the cause of


gynaecomastia.
D Increased HCG production

Increased HCG production is incorrect. Liver disease is not a cause of increased HCG
production, although neoplasms that produce HCG may themselves lead to gynaecomastia.

E Increased testosterone catabolism

Increased testosterone catabolism is incorrect. Catabolism of steroid hormones is decreased


in patients with liver disease.
40977

Rate this question:

Next Question

Previous Question Tag Question

Feedback End Review

Difficulty: Average

Peer Responses %

Q. Answered Flagged

Q1

Q2

Q3

Q4

Q5

Q6
Q. Answered Flagged

Q7

Q8

0:00:07/03:00:00

A 70-year-old woman comes to the Respiratory Clinic for assessment of COPD. She has
smoked 20 cigarettes per day for 50 years. Over the past 6 months she has experienced a
progressive fall off in her exercise tolerance from 2 miles to only 300 metres, and marked
shortness of breath on climbing the stairs. This has been poorly responsive to a Salbutamol
inhaler.

A Normal FEF 25-75, markedly reduced FVC

B Normal expiratory study, flattened inspiratory loop

C Reduced FEF 25-75, slightly reduced FVC, normal inspiratory loop

D Reduced FEF 25-75, normal FVC, flattened inspiratory loop

E Reduced FEF 25-75, reduced FVC, normal inspiratory loop

Explanation 

C Reduced FEF 25-75, slightly reduced FVC, normal inspiratory loop

A typical patient with COPD has a concave appearance to the expiratory portion of the flow
volume loop, FEV1 and FEF 25-75 are reduced, FVC is normally slightly reduced due to air
trapping.

A Normal FEF 25-75, markedly reduced FVC

A normal FEV 25-75 is not consistent with small airways obstruction.

B Normal expiratory study, flattened inspiratory loop

This is consistent with large airway obstruction.

D Reduced FEF 25-75, normal FVC, flattened inspiratory loop

The inspiratory loop is normal in small airways obstruction.


E Reduced FEF 25-75, reduced FVC, normal inspiratory loop

This is consistent with a mixed obstructive / restrictive picture.

41037

Rate this question:

Next Question

Previous Question Tag Question

Feedback End Review

Difficulty: Average

Peer Responses %

Q. Answered Flagged

Q1

Q2

Q3

Q4

Q5

Q6

Q7

Q8

Q9

0:00:07/03:00:00

A 22-year-old assistant in a pharmacy is admitted for the third occasion in 6 months with a
collapse whilst working in the shop. She says she feels faint and lethargic all the time, and
denies any significant past medical history. Her only medication is the combined oral
contraceptive pill. Her BP is 95/60 mmHg, pulse is 80/min and regular. Her BMI is 23. She is
1.72m tall.

Investigations:

Hb 12.0 g/dl

WCC 6.4 x109/l

PLT 171 x109/l

Na+ 138 mmol/l

K+ 3.0 mmol/l

Bicarbonate 33 mmol/l

Calcium 2.6 mmol/l

Magnesium 0.6 mmol/l

Glucose 5.0 mmol/l

A Bartter’s syndrome

B Conn’s syndrome

C Cushing’s syndrome

D Liddle’s syndrome

E Gitelman’s syndrome

Explanation 

E Gitelman’s syndrome
The presentation in adulthood, with syncopal episodes, hypokalaemia, and
hypomagnesaemia fits well with a diagnosis of GS. Calcium excretion is decreased, whereas
in Bartter’s it is normal or increased, (a potential differentiator). In this situation abuse of
thiazide diuretics is a possible alternative diagnosis, and with appropriate consent from the
patient, urine diuretic screen is normally considered. NSAIDs, potassium and magnesium
supplementation are all potential therapeutic interventions.

A Bartter’s syndrome

Bartter’s syndrome is incorrect. Bartter’s usually presents in early childhood with failure to
thrive. Although it is associated with hypokalaemia, it is associated with normal or increased
calcium excretion, and normal magnesium, (unlike the picture here).

B Conn’s syndrome

Conn’s syndrome is incorrect. Conn’s syndrome is associated with hypertension.

C Cushing’s syndrome

Cushing’s syndrome is incorrect. Cushing’s is associated with hypertension, weight gain and
impaired glucose tolerance.

D Liddle’s syndrome

Liddle’s syndrome is incorrect. Liddle’s syndrome is a rare inherited form of hypertension


characterised by severe early-onset hypertension associated with decreased plasmatic
levels of potassium, renin and aldosterone.
41022

Rate this question:

Next Question

Previous Question Tag Question

Feedback End Review

Difficulty: Average

Peer Responses %
Q. Answered Flagged

Q1

Q2

Q3

Q4

Q5

Q6

Q7

Q8

Q9

0:00:07/03:00:00

A 38-year-old man who had coeliac disease diagnosed at the age of 12, comes to the
Emergency Department for review with increasing abdominal pain, bloating, nausea and
vomiting for the past 24 hours. He admits that over the past few months he has suffered
from worsening coeliac symptoms, with diarrhoea, night sweats and gradual loss in weight.
He takes no regular medications. On examination his BP is 115/85 mmHg; pulse is 80/min
and regular. He is thin with a BMI is 19, abdomen is distended with central, palpable
distended small bowel. Bowel sounds are sparse.

Investigations:

Hb 9.4 g/dl

WCC 5.1 x109/l

PLT 123 x109/l

Na+ 135 mmol/l

K+ 4.3 mmol/l

Creatinine 95 micromol/l

Ca2+ 2.1 mmol/l

ALP 185 U/l

ALT 132 U/l

ESR 75 mm/1st hour

Abdominal X-ray small bowel obstruction

A Bacterial overgrowth syndrome

B B cell lymphoma

C Caecal carcinoma

D Duodenal carcinoma

E Enteropathy associated T cell lymphoma


Explanation 

E Enteropathy associated T cell lymphoma

The presentation here is consistent with small bowel obstruction, and against a background
of long-standing coeliac disease, coupled with abdominal pain and night sweats, T cell
lymphoma must be considered as an underlying cause. Diagnosis is primarily confirmed by
small bowel biopsy specimen analysis, unfortunately these are aggressive lymphomas and
outcome is often poor, despite intensive chemotherapy, (median survival only 10 months).

A Bacterial overgrowth syndrome

Bacterial overgrowth syndrome is incorrect. Although bacterial overgrowth is associated with


symptoms of bloating, it is not associated with small bowel obstruction.

B B cell lymphoma

B cell lymphoma is incorrect. It is normally T cell lymphomas that are associated with long
standing coeliac disease.

C Caecal carcinoma

Caecal carcinoma is incorrect. Coeliac disease is not associated with increased risk of colon
cancer, and the poor control of coeliac symptoms coupled with “B symptoms”, abdominal
pain and night sweats, fits better with a lymphoma.

D Duodenal carcinoma

Duodenal carcinoma is incorrect. It is a rare carcinoma, usually seen in association with


familial GI tumour syndromes.
40971

Rate this question:

Next Question

Previous Question Tag Question

Feedback End Review

Difficulty: Average

Peer Responses %
Q. Answered Flagged

Q1

Q2

Q3

Q4

Q5

Q6

Q7

Q8

Q9

0:00:07/03:00:00

An 80-year-old man comes to the clinic for review. He takes a number of vitamin
supplements, statins, aspirin and anti-hypertensive medications. He is interested in adding to
his medications to enable him to keep healthy for another 30-40 years. The doctor explains
that he cannot live forever because of the Hayflick limit.

A The maximum amount of time over which a cell is responsive to growth stimuli

B The maximum amount of peptides that can be produced by a cell

C The maximum number of times a human cell can divide

D The maximum number of mutations in DNA that can be repaired

E The maximum amount of DNA within a cell

Explanation 

C The maximum number of times a human cell can divide

The Hayflick limit is the maximum number of times that a human cell can divide until cell
division stops. This is thought to be related to telomere shortening, and once it is reached,
cells enter senescence. Hayflick demonstrated his principle using foetal cells in culture, and
estimated the limit to be between 40 and 60. The limit is thought to restrict genetic
instability, and may impact on risk of cancer.

A The maximum amount of time over which a cell is responsive to growth stimuli

The maximum amount of time over which a cell is responsive to growth stimuli is incorrect. A
cell may not require an external growth stimulus to divide, and this implies an indirect
mechanism in controlling cell division.

B The maximum amount of peptides that can be produced by a cell


The maximum amount of peptides that can be produced by a cell is incorrect, although
repeated cell division may result in accumulated mutations which affect a cells ability to
produce peptides.

D The maximum number of mutations in DNA that can be repaired

The maximum number of mutations in DNA that can be repaired is incorrect, although
accumulated mutations may result in a apoptosis or potentially cancerous transformation.

E The maximum amount of DNA within a cell

The maximum amount of DNA within the cell is incorrect. The limit on division is not
related to the amount of DNA within a cell.
41053

Rate this question:

Next Question

Previous Question Tag Question

Feedback End Review

Difficulty: Average

Peer Responses %

Q. Answered Flagged

Q1

Q2

Q3
Q. Answered Flagged

Q4

Q5

Q6

Q7

0:00:07/03:00:00

An 18-year-old student presents to the Emergency Department with a severe sore throat
and fever. He has been unwell for the past 48hrs, and his symptoms have responded very
poorly to both ibuprofen and paracetamol taken together. On examination his BP is 105/78
mmHg, pulse is 85/min and regular, he is pyrexial 38.5°C. There is cervical and
submandibular lymphadenopathy, and exudative pharyngitis with severe tonsillar
inflammation.

Investigations:

Hb 13.4 g/dl

WCC 8.4 x 109/l (large atypical lymphocytes seen on blood film)

PLT 238 x 109/l

Na+ 137 mmol/l

K+ 4.5 mmol/l

Creatinine 100 micromol/l

CRP 185 mg/l

Monospot negative

A Acute lymphocytic leukaemia

B Acute myeloid leukaemia

C Cytomegalovirus

D Epstein Barr virus

E Streptococcus pyogenes infection

Explanation 

D Epstein Barr virus


The large atypical lymphocytes, in a patient with severe pharyngitis, are most likely to be
linked to acute EBV infection, resulting in the severe pharyngitis and lymphadenopathy seen
here. The fact that the Monospot is negative is not a deterrent to confirming the diagnosis, as
it may be falsely negative for 25% of patients in the first week of infection. Splenomegaly is
seen in 50-60% of patients with EBV, although rupture is rare, occurring in 1-2 / 1000 cases of
EBV. Management of EBV is supportive, with use of anti-pyretics and instruction to maintain
adequate hydration.

A Acute lymphocytic leukaemia

Acute lymphocytic leukaemia is incorrect. Circulating blasts are seen in AL, and the disease
commonly presents with symptoms of rapidly progressive anaemia, fevers, anorexia and
weight loss.

B Acute myeloid leukaemia

Acute myeloid leukaemia is incorrect. In AML myeloid pre-cursors are seen in peripheral
blood.

C Cytomegalovirus

Cytomegalovirus is incorrect. Although the Monospot is negative, the presentation is most


consistent with EBV rather than CMV.

E Streptococcus pyogenes infection

Streptococcus pyogenes is incorrect. The picture fits better with EBV infection due to the
presence of atypical lymphocytes.
41007

Rate this question:

Next Question

Previous Question Tag Question

Feedback End Review

Difficulty: Average

Peer Responses %
Q. Answered Flagged

Q1

Q2

Q3

Q4

Q5

Q6

Q7

Q8

Q9

0:00:07/03:00:00

A 33-year-old man with ulcerative pancolitis was seen. His symptoms were worsening despite
intravenous hydrocortisone.

A Ciclosporin is indicated to induce remission

B Cytomegalovirus is a common cause of non-responsive colitis

C Methotrexate is indicated to induce remission

D Non-steroidal anti-inflammatory drugs (NSAIDs) are a useful adjunct to therapy.

E Surgery is contraindicated

Explanation 

A Ciclosporin is indicated to induce remission

The options for treating acute severe ulcerative colitis that is refractory to intravenous
hydrocortisone are medical or surgical. The major options for medical therapy are either
intravenous ciclosporin or infliximab; the surgical option is a subtotal colectomy.

Other immunosuppressive medications that have a role in treating chronic ulcerative colitis –
including the thiopurines, azathioprine and 6-mercaptopurine – take several weeks before
they start to produce benefit. As such, unlike ciclosporin or infliximab, these may not be used
in people with refractory acute severe colitis to induce remission quickly. However, it is not
an unusual scenario for a patient with refractory acute severe colitis to start ciclosporin and a
thiopurine together, with the hope that the ciclosporin will quickly induce remission, and
serve as a ‘bridge’ until the effect of the thiopurine starts to appear over the next few weeks.

B Cytomegalovirus is a common cause of non-responsive colitis

Cytomegalovirus is a common cause of non-responsive colitis is incorrect. Cytomegalovirus


is in fact an uncommon cause of non-responsive colitis, but is important to consider because
anti-viral treatment is available.

C Methotrexate is indicated to induce remission


Methotrexate is indicated to induce remission is incorrect. Methotrexate has a limited role in
the treatment of chronic Crohn’s disease, but no generally recognised role in the treatment
of ulcerative colitis, either in acute or chronic disease.

D Non-steroidal anti-inflammatory drugs (NSAIDs) are a useful adjunct to therapy.

Non-steroidal anti-inflammatory drugs (NSAIDs) are a useful adjunct to therapy is


incorrect. Non-steroidal anti-inflammatory drugs (NSAIDs) cause flares of inflammatory
bowel disease, rather than helping to induce remission.

E Surgery is contraindicated

Surgery is contraindicated is incorrect. Although these patients may be very unwell and
potentially are risky surgical candidates, it is also possible that colectomy will be life-saving,
and surgery is therefore not contraindicated.
45001

Rate this question:

Next Question

Previous Question Tag Question

Feedback End Review

Difficulty: Average

Peer Responses %

Q. Answered Flagged

Q1

Q2
Q. Answered Flagged

Q3

Q4

Q5

Q6

Q7

Q8 
0:00:07/03:00:00

A 22-year-old woman presents to the Emergency Department with an extensive scaling


erythematous rash. She presented to the GP some 2-3 weeks earlier with an upper
respiratory tract infection for which she was prescribed a course of Clarithromycin as she is
penicillin allergic. On examination her BP is 105/80 mmHg, pulse is 70/min and regular, and
she is apyrexial. There are multiple salmon pink patches with superficial scaling, each
approximately 1cm in diameter. Routine bloods are unremarkable.

A Erythema multiforme

B Guttate psoriasis

C Lichen planus

D Pityriasis rosea

E Seborrhoeic eczema

Explanation 

B Guttate psoriasis

The presentation with an extensive scaling erythematous rash, some 2-3 weeks after a
respiratory tract infection is typical of guttate psoriasis. The pathogenesis of guttate
psoriasis is complex, although given the proximity to a recent respiratory tract infection, a T
cell driven autoimmune response is suspected. The rash usually fades spontaneously over a
period of a few weeks, although sunlight exposure / UVB phototherapy can accelerate
recovery.

A Erythema multiforme

Erythema multiforme is incorrect. It is associated with multiple erythematous lesions,


although lesions are potentially blistering and have a typical target like appearance.

C Lichen planus
Lichen planus is incorrect. Lichen planus is typically associated with much smaller papules,
appearing initially on the flexor surfaces of the wrists.

D Pityriasis rosea

Pityriasis rosea is incorrect. Pityriasis rosea is associated with a larger patch of erythema,
known as a herald patch, which appears initially on the abdomen.

E Seborrhoeic eczema

Seborrhoeic eczema is incorrect. This commonly leads to an erythematous scaling rash


affecting the scalp initially.
40962

Rate this question:

Next Question

Previous Question Tag Question

Feedback End Review

Difficulty: Average

Peer Responses %

Q. Answered Flagged

Q1

Q2

Q3

Q4
Q. Answered Flagged

Q5

Q6

Q7

Q8 
0:00:07/03:00:00

You are reviewing a 26-year-old man who is recovering from a motor bike accident with
severe injuries sustained to his left leg. He has flaccid weakness affecting plantar flexion of
his left foot.

A Gastrocnemius

B Flexor hallucis longus

C Peroneus longus

D Tibialis anterior

E Tibialis posterior

Explanation 

A Gastrocnemius

The muscle is involved in plantar flexion. Hypertrophy of the gastrocnemius in athletes is


recognised to lead to popliteal entrapment syndrome, and hypertrophy of the
gastrocnemius is recognised in inherited muscular dystrophies. It is innervated by the S1,S2
nerve roots, (tibial nerve).

B Flexor hallucis longus

Flexor hallucis longus is incorrect. This muscle is responsible for flexion of the great toe, is
innervated by the tibial nerve, (S2,S3).

C Peroneus longus

Peroneus longus is incorrect. Peroneus longus is innervated by S1, and is responsible for
eversion of the foot.

D Tibialis anterior
Tibialis anterior is incorrect. Tibialis anterior is responsible for ankle dorsiflexion, and is
innervated by the deep peroneal nerve, (L4,5).

E Tibialis posterior

Tibialis posterior is incorrect. Tibialis posterior is involved in inversion and adduction of the
foot, and is innervated by the tibial nerve, (L4,5).
41010

Rate this question:

Next Question

Previous Question Tag Question

Feedback End Review

Difficulty: Average

Peer Responses %

Q. Answered Flagged

Q1

Q2

Q3

Q4

Q5

Q6

Q7
Q. Answered Flagged

Q8
0:00:07/03:00:00

A 47-year-old woman presents to the Cancer Clinic for review. She is concerned as two of
her first-degree relatives died of colon cancer at the age of 64 and 62. A recent private
colonoscopy has been unremarkable. She asks about preventative measures to reduce her
future risk of developing the disease.

A Aspirin

B Hormone replacement therapy

C Ramipril

D Simvastatin

E Vitamin A

Explanation 

A Aspirin

A range of studies suggest between a 20 and 40% reduction in the incidence of colon cancer
in patients prescribed Aspirin and other NSAIDs. The effect is related both to duration of
therapy and appears to be dose related. There is a suggestion that selective COX-2 inhibitors
may be more effective, although cardiovascular adverse events associated with their use
have precluded wide spread use.

B Hormone replacement therapy

Hormone replacement therapy is incorrect. Although early studies suggested a reduction in


incidence, HRT appeared to be associated with presentation with more advanced tumours
initially.

C Ramipril

Ramipril is incorrect. Although angiotensin II has been shown in animal models to be


associated with development of cancer, intervention has not been shown to impact on risk.
D Simvastatin

Simvastatin is incorrect. There is no evidence that Simvastatin reduces risk of colon cancer
development.

E Vitamin A

Vitamin A is incorrect. Vitamin A supplements have not been shown to be protective


against the development of cancer, in some patients they have even been shown to
increase the risk of lung cancer.
41063

Rate this question:

Next Question

Previous Question Tag Question

Feedback End Review

Difficulty: Average

Peer Responses %

Q. Answered Flagged

Q1

Q2

Q3

Q4

Q5
Q. Answered Flagged

Q6

Q7


Q8
0:00:07/03:00:00

A 25-year-old woman who is taking Azathioprine for Crohn’s disease comes to the clinic for
review. She is 8 weeks pregnant with her first child. Her Crohn’s is currently quiescent, and
she opens her bowels some 3 times per day with semi-solid or normal stools. She has had
one previous bowel resection for Crohn’s some 3 years earlier. On examination her BP is
110/80 mmHg, pulse is 70/min and regular. She has a midline scar on her abdomen, her
BMI is 22.

Investigations:

Hb 11.3 g/dl

WCC 8.0 x109/l

PLT 171 x 109/l

Na+ 137 mmol/l

K+ 4.5 mmol/l

Creatinine 90 micromol/l

ESR 18 mm/1st hour

Albumin 32 g/l

ALT 23 U/l

Calcium 2.2 mmol/l

A Add Prednisolone

B Continue Azathioprine

C Switch to Ciclosporin

D Switch to Methotrexate

E Switch to Sulphasalazine

Explanation 
B Continue Azathioprine

According to guidelines from the British Society of Gastroenterology, (BSG). “Azathioprine


should in general be continued during pregnancy, as the risks to the fetus from disease
activity appear to be greater than continued therapy. Babies born to mothers on Azathioprine
may be lighter than normal”. As such, given her disease is currently under control, it is most
appropriate to continue Azathioprine at the current dose.

A Add Prednisolone

Add Prednisolone is incorrect. Although the benefit risk for use of corticosteroids is positive
during an acute exacerbation, chronic use in pregnancy is associated with significant adverse
effects of therapy, including weight gain, impaired glucose tolerance, loss of bone mineral
density and potential macrosomia.

C Switch to Ciclosporin

Switch to Ciclosporin is incorrect. Ciclosporin use in pregnancy is associated with a


similar profile to Azathioprine, with the potential for low birthweight or premature
delivery, hence there is no reason to switch.

D Switch to Methotrexate

Switch to Methotrexate is incorrect. Methotrexate is teratogenic and contra-indicated, and


must not be used in this situation.

E Switch to Sulphasalazine

Switch to Sulphsalazine is incorrect. Sulphasalazine may be associated with haemolytic


anaemia in the offspring, and hence is best avoided in this situation.
41031

Rate this question:

Next Question

Previous Question Tag Question

Feedback End Review

Difficulty: Average

Peer Responses %
Q. Answered Flagged

Q1

Q2

Q3

Q4

Q5

Q6

Q7

Q8

Q9

0:00:07/03:00:00

A 45-year-old woman presents to the Emergency Department with gross haematuria that has
persisted over the last 3 days. She has also become progressively more short of breath such
that she is unable to climb the stairs. She attended her GP some 2 weeks earlier with dysuria
and fevers. However despite being prescribed antibiotics, her symptoms have not improved.
On examination her BP is 155/85 mmHg; pulse is 95/min and regular. Her temperature is
38.0°C and she looks pale. There are bilateral inspiratory crackles over the lung bases.

Investigations:

Hb 9.0 g/dl

WCC 12.1 x 109/l

PLT 142 x 109/l

Na+ 138 mmol/l

K+ 5.4 mmol/l

Creatinine 205 micromol/l

ESR 40 mm/1st hour

CXR bilateral pulmonary infiltrates

Urine blood +++, protein ++, nitrites -

A Goodpasture’s syndrome

B Polyarteritis nodosa

C Renal cell carcinoma

D Urinary tract infection

E Wegener’s granulomatosis

Explanation 
A Goodpasture’s syndrome

GS is associated with rapidly progressive glomerulonephritis and pulmonary haemorrhage,


which may well account for the increased shortness of breath over the past few days and the
pulmonary infiltrates identified on CXR. Anaemia is almost certainly related to intrapulmonary
bleeding. Percutaneous renal biopsy and transbronchial biopsy are both employed in
confirming the diagnosis, and high dose corticosteroids coupled with Cyclophosphamide are
the initial management of choice.

B Polyarteritis nodosa

Polyarteritis nodosa is incorrect. This presents with a more chronic course and is commonly
associated with symptoms of peripheral angina and mesenteric angina.

C Renal cell carcinoma

Renal cell carcinoma is incorrect. It would not be associated with such rapid deterioration
as is seen here, and pulmonary infiltrates would be unusual versus discrete metastases.

D Urinary tract infection

Urinary tract infection is incorrect. The failure of response to antibiotics, coupled with a
nitrite negative urine specimen makes this unlikely.

E Wegener’s granulomatosis

Wegener’s granulomatosis is incorrect. This is associated with a more chronic deterioration


and usually with evidence of midline granulomatous disease, including sinusitis.
41016

Rate this question:

Next Question

Previous Question Tag Question

Feedback End Review

Difficulty: Average

Peer Responses %
Q. Answered Flagged

Q1

Q2

Q3

Q4

Q5

Q6

Q7

Q8

Q9

0:00:07/03:00:00

A 28-year-old man is admitted to the Emergency Department having been found collapsed
at the paint sprayers where he works, having taken an overdose of methanol, used in the
workshop for diluting paint. Two empty 250ml containers were found next to him, it is
unclear how much he took. On examination in the Emergency Department his GCS is 11, he
is intoxicated and smells of alcohol. His chest appears clear and abdomen is soft, although
he is tender in the epigastrium. Investigations reveal an evolving metabolic acidosis. You
plan to start him on Fomepizole.

A Agonist of alcohol dehydrogenase

B Competitive inhibitor of alcohol dehydrogenase

C Competitive inhibitor of aldehyde dehydrogenase

D Non-competitive inhibitor of alcohol dehydrogenase

E Partial agonist of aldehyde dehydrogenase

Explanation 

B Competitive inhibitor of alcohol dehydrogenase

Fomepizole, or 4-methyl pyrazole, is used as an antidote for ethylene glycol and for
methanol poisoning. By inhibiting alcohol dehydrogenase it prevents accumulation of
formaldehyde, which is then further metabolised to formic acid leading to acidosis and
potential ocular toxicity. It is usually continued until ethylene glycol or methanol
concentration is undetectable or ethylene glycol or methanol concentration is less than 50
mg/L AND acidosis and signs of systemic toxicity have resolved.

A Agonist of alcohol dehydrogenase

Agonist of alcohol dehydrogenase is incorrect. An agonist would accelerate accumulation of


formic acid and lead more rapidly to toxicity.

C Competitive inhibitor of aldehyde dehydrogenase


Competitive inhibitor of aldehyde dehydrogenase is incorrect. Aldehyde dehydrogenase
inhibitors include Disulfiram, which is used to exacerbate “hangover” symptoms of alcohol
use in patients trying to abstain from alcohol.

D Non-competitive inhibitor of alcohol dehydrogenase

Non-competitive inhibitor of alcohol dehydrogenase is incorrect. Fomepizole competes


directly with alcohols for binding to alcohol dehydrogenase.

E Partial agonist of aldehyde dehydrogenase

Partial agonist of aldehyde dehydrogenase is incorrect. Agonism of aldehyde


dehydrogenase would accelerate formation of formic acid and is therefore not a treatment
step for methanol poisoning.
41161

Rate this question:

Next Question

Previous Question Tag Question

Feedback End Review

Difficulty: Average

Peer Responses %

Q. Answered Flagged

Q1

Q2

Q3
Q. Answered Flagged

Q4

Q5

Q6

Q7

Q8 
0:00:07/03:00:00

A 33-year-old woman presents with polydipsia and polyuria. Her symptoms started soon
after a road traffic accident 6 months ago. She has no past medical history of note apart
from a period of depression in her 20s. Her blood pressure is 120/80 mmHg with no postural
drop on moving from a lying to standing position. Her daily urinary output is 6–8 l. Blood
tests show: sodium 130 mmol/l (normal range 137–144 mmol/l); potassium 3.5 mmol/l (3.5–
4.9 mmol/l); urea 6 mmol/l (2.5–7.5 mmol/l); and glucose 4 mmol/l (3–6 mmol/l). Plasma
osmolality is 268 mosmol/l (278–305 mosmol/l) and urine osmolality is 50 mosmol/l (350–
1000 mosmol/l).

A Cranial diabetes insipidus

B Diuretic phase of acute renal failure

C Nephrogenic diabetes insipidus

D Psychogenic polydipsia

E Syndrome of inappropriate antidiuretic hormone secretion

Explanation 

D Psychogenic polydipsia

In psychogenic polydipsia, there is excessive water intake and so the serum sodium, plasma
osmolality and urine osmolality will all be decreased. The low urine and plasma osmolality
seen here, coupled with low sodium, are therefore supportive of the diagnosis. Her previous
history of depression, although not a strong pointer to psychogenic polydipsia, is at least
supportive of the diagnosis.

A Cranial diabetes insipidus

Cranial diabetes insipidus is incorrect. In diabetes insipidus, whether cranial or


nephrogenic, there will be an increase in plasma osmolality and serum sodium levels as
water is lost from the body. Postural hypotension is likely to be seen.
B Diuretic phase of acute renal failure

Diuretic phase of acute renal failure is incorrect. There is no trigger to suggest renal
failure, and in the diuretic phase of acute renal failure, the serum sodium level will be
increased.

C Nephrogenic diabetes insipidus

Nephrogenic diabetes insipidus is incorrect. Nephrogenic diabetes insipidus is associated


with an increase in plasma osmolality, elevated serum sodium and postural hypotension.

E Syndrome of inappropriate antidiuretic hormone secretion

Syndrome of inappropriate antidiuretic hormone secretion is incorrect. The syndrome of


inappropriate secretion of antidiuretic hormone (SIADH) leads to retention of water and
hyponatraemia. Plasma osmolality will be decreased as well. However, the urine osmolality
will be raised and can even be higher than the plasma osmolality.
49984

Rate this question:

Next Question

Previous Question Tag Question

Feedback End Review

Difficulty: Average

Peer Responses %

Q. Answered Flagged

Q1
Q. Answered Flagged

Q2

Q3

Q4

Q5

Q6

Q7

Q8
0:00:07/03:00:00

A 34-year-old man presents with loss of pain and temperature sensation over his shoulders
and upper arms, marginally worse on the right hand side compared with the left. History of
a rear shunt car accident at approximately 50 MPH some 9 months earlier is noted, and he
has asthma for which he takes a Seretide inhaler. On examination his BP is 110/82 mmHg;
pulse is 75/min and regular. There is loss of pain and temperature sensation in a shawl like
distribution over his shoulders in particular, fine touch and proprioception is intact. Routine
bloods are entirely normal.

A Cervical radiculopathy

B Limb girdle muscular dystrophy

C Motor neurone disease

D Spondylolisthesis

E Syringomyelia

Explanation 

E Syringomyelia

The suspicion, given the distribution of sensory loss seen here, is of post-traumatic
syringomyelia, which may be seen after trauma such as his recent motor vehicle accident.
MRI is the investigation of choice to confirm the presence of a syrinx. The patient should be
reviewed by a neurosurgeon to determine whether intervention may reduce progression of
his symptoms.

A Cervical radiculopathy

Cervical radiculopathy is incorrect. There is a relative absence of neck pain, and dissociated
sensory loss is not a feature.

B Limb girdle muscular dystrophy


Limb girdle muscular dystrophy is incorrect. Limb girdle muscular dystrophy leads to
predominantly proximal muscle weakness rather than the sensory symptoms seen here.

C Motor neurone disease

Motor neurone disease is incorrect. Motor neurone disease presents with muscle weakness
rather than the sensory symptoms seen here.

D Spondylolisthesis

Spondylolisthesis is incorrect. The predominant feature of spondylolisthesis is of neck pain


and a mixed picture of motor and sensory loss, not seen here.
41084

Rate this question:

Next Question

Previous Question Tag Question

Feedback End Review

Difficulty: Average

Peer Responses %

Q. Answered Flagged

Q1

Q2

Q3

Q4
Q. Answered Flagged

Q5

Q6

Q7

Q8 
0:00:07/03:00:00

An Australian backpacker returned to the UK 2 months ago following an extended trip to


East and southern Africa. He now presents with fever, giant urticaria, a headache and
bloody diarrhoea. Blood tests are normal other than an eosinophil count of 7.2 x 109/l.

A Ascaris lumbricoides

B Entamoeba histolytica

C Giardia lamblia

D Schistosoma mansoni

E Strongyloides stercoralis

Explanation 

D Schistosoma mansoni

Katayama fever develops some 6 weeks to 3 months after infection with Schistosoma
spp., caused by an immune reaction to the highly antigenic schistosomal eggs.
It is characterised by fever, hepatosplenomegaly, urticaria and lymphadenopathy.
The present patient also has a schistosomal colitis, which can be very severe in
those without previous exposure.
Diagnosis is made on serology and by rectal biopsy (looking for the eggs).
Corticosteroids may be helpful in reducing inflammation in Katayama fever; the
underlying infection is treated with praziquantel.

A Ascaris lumbricoides

Ascaris lumbricoides is incorrect. Ascaris spp. can cause eosinophilia, and infected
individuals may develop Löffler syndrome (also known as pulmonary eosinophilia),
characterised by fever, urticaria, wheeze and dry cough, due to migration of the worm
through the lungs,
typically 10–16 days after infection. The delay of 2 months, in this case, makes this a less
likely diagnosis.

B Entamoeba histolytica

Entamoeba histolytica is incorrect. E. histolytica can cause a chronic colitis, but one would
not expect to see eosinophilia or urticaria.

C Giardia lamblia

Giardia lamblia is incorrect. Giardiasis typically presents with gradual-onset non-bloody


diarrhoea, with bloating and flatulence; an eosinophilia and urticarial would not be expected.

E Strongyloides stercoralis

Strongyloides stercoralis is incorrect. S. stercoralis infection is likely to cause an eosinophilia,


but this degree of systemic upset, particularly the bloody diarrhoea, would be unlikely in an
immunocompetent individual. Strongyloides may cause Löffler syndrome

Further reading

Checkley A M, Chiodini P L, Dockrell D H, Bates I, Thwaites G E, Booth H L, et al. 2010.


Eosinophilia in returning travellers and migrants from the tropics: UK recommendations for
investigation and initial management. Journal of Infection, 60, 1, 1–20.
http://www.journalofinfection.com/article/S0163-4453%2809%2900360-0/fulltext
(http://www.journalofinfection.com/article/S0163-4453%2809%2900360-0/fulltext)

Johnston V, Stockley J M, Dockrell D, Warrell D, Bailey R, Pasvol G, et al. 2009. Fever in


returned travellers presenting in the United Kingdom: Recommendations for investigation
and initial management. Journal of Infection, 59, 1–18.
http://www.journalofinfection.com/article/S0163-4453(09)00154-6/fulltext
(http://www.journalofinfection.com/article/S0163-4453(09)00154-6/fulltext)
49803

Rate this question:

Next Question

Previous Question Tag Question

Feedback End Review

Difficulty: Difficult

Peer Responses %
Q. Answered Flagged

Q1

Q2

Q3

Q4

Q5

Q6

Q7

Q8

Q9

0:00:07/03:00:00

A 60-year-old man is referred after a chest X-ray suggests interstitial lung disease. You
proceed to bronchoscopy with transbronchial lung biopsy to try and make a definitive
histological diagnosis.

A Cryptogenic organising pneumonia

B Extrinsic allergic alveolitis

C Idiopathic pulmonary fibrosis

D Lymphangitis carcinomatosa

E Sarcoidosis

Explanation 

C Idiopathic pulmonary fibrosis

IPF (previously known as cryptogenic fibrosing alveolitis) can be a patchy disease and
transbronchial bronchoscopy might simply reveal established fibrosis with no particular
aetiology suggested. If a biopsy is required in IPF then a surgical lung biopsy, guided by high-
resolution computed tomography, will provide better specimens.

A Cryptogenic organising pneumonia

Cryptogenic organising pneumonia is incorrect. Cryptogenic organising pneumonia can be


diagnosed from transbronchial biopsy by the presence of organising pneumonia on
histological examination.

B Extrinsic allergic alveolitis

Extrinsic allergic alveolitis is incorrect. Extrinsic allergic alveolitis can be diagnosed from
transbronchial biopsy and would be suggested by a finding of granulomas.

D Lymphangitis carcinomatosa
Lymphangitis carcinomatosa is incorrect. Lymphangitis carcinomatosa can be diagnosed
from transbronchial biopsy by the presence by malignant cells with diffuse bronchial
and peribronchial lymphatic involvement.

E Sarcoidosis

Sarcoidosis is incorrect. Sarcoidosis can be diagnosed from transbronchial biopsy and would
be suggested by a finding of granulomas.
43728

Rate this question:

Next Question

Previous Question Tag Question

Feedback End Review

Difficulty: Average

Peer Responses %

Q. Answered Flagged

Q1

Q2

Q3

Q4

Q5

Q6
Q. Answered Flagged

Q7

Q8
0:00:07/03:00:00

A 21-year-old man comes to the Emergency Department complaining that his neighbour is
controlling his thoughts. He says that broadcasts from a radio mast in the neighbour’s garden
are penetrating his brain despite him putting up silver foil against all of his downstairs
windows. He is a regular user of cannabis. When you chat to him in the department he tells
you he is running from the secret service, which have been chasing him since he returned
from Iraq. There is no past medical history of note and he appears not to have attended the
department before. Physical examination and routine bloods are unremarkable. You
prescribe Risperidone.

A 12 hours

B 24 hours

C 3 days

D 1 week

E 4-6 weeks

Explanation 

E 4-6 weeks

This man has symptoms of acute schizophrenia. With respect to giving advice about
symptom resolution, it is appropriate to be conservative, as some patients may take 4-6
weeks for their psychosis to resolve. The most rapid improvement is usually seen during the
first two weeks of therapy. There is little difference between anti-psychotics; Clozapine is
potentially marginally more efficacious, although it is associated with agranulocytosis, and
is therefore reserved for patients with resistant symptoms.

A 12 hours

12 hours is incorrect. Although anti-psychotics may reduce agitation, they will not lead to
symptom resolution in this time period.
B 24 hours

24 hours is incorrect. As previously described, symptoms are only likely to be completely


resolved over the 4-6 week time period.

C 3 days

3 days is incorrect, although some improvement in psychosis may be seen by day 3.

D 1 week

1 week is incorrect, although the fastest improvement in symptoms is seen over the first 1-2
weeks of therapy.
40963

Rate this question:

Next Question

Previous Question Tag Question

Feedback End Review

Difficulty: Average

Peer Responses %

Q. Answered Flagged

Q1

Q2

Q3
Q. Answered Flagged

Q4

Q5

Q6

Q7

Q8 
0:00:07/03:00:00

A 30-year-old woman with systemic lupus erythematosus (SLE) is eager to find out if she has
any predisposing factors for the disease.

A Dizygotic twin of a patient with SLE

B First-degree relative of a patient with SLE

C Monozygotic twin of a patient with SLE

D Positive HLA-B8

E Positive HLA–DR2

Explanation 

C Monozygotic twin of a patient with SLE

There is a higher concordance rate in monozygotic twins (up to 25%) compared to dizygotic
twins (up to 3%) in systemic lupus erythematosus (SLE).

A Dizygotic twin of a patient with SLE

Dizygotic twin of a patient with SLE is incorrect. Dizygotic twins have a concordance rate of
up to 3% in SLE, whereas monozgoyic twins can have a concordance rate of up to 25%.

B First-degree relative of a patient with SLE

First-degree relative of a patient with SLE is incorrect. First-degree relatives have a 3%


chance of developing the disease.

D Positive HLA-B8

Positive HLA-B8 is incorrect, although there is an increased frequency of HLA B8 in white


SLE populations.
E Positive HLA–DR2

Positive HLA–DR2 is incorrect, although here is a stronger association with HLA DR2 in
Japanese people with lupus.
48848

Rate this question:

End Session

Previous Question Tag Question

Feedback End Review

Difficulty: Average

Peer Responses %

Q. Answered Flagged

Q1

Q2

Q3

Q4

Q5

Q6

Q7

Q8

Q9

You might also like